You are on page 1of 120

Chapter 1:

The Nature of Mathematics


Introduction
M
athematics has been part of human’s everyday life. It is everywhere. It is on
the things we create and admire, in the activities we do, in the decisions we make.
Mathematics is present in everything that surrounds us. It could be seen on the
patterns and sequences exhibited by nature from its landscapes, plants and animals,
including humans to series of events and phenomena.
Recognizing these patterns gives us understanding on how nature works and
helps create things that human may benefit from. Through this, advancement of
science and technology could be done. Mathematics could help predict the behavior
of nature especially the naturally-occurring phenomena and create intervention to
safeguard the lives of human. This could provide necessary answers to the questions
that puzzles humankind that may create a much better world.
In this chapter, we will look at mathematics as a science of pattern and order
to encourage you to go beyond the typical understanding of mathematics as merely a
set of formulas but as a source of aesthetics in patterns of nature and a rich
language in itself governed by logic and reasoning.

Learning Outcomes:

At the end of this chapter, you are expected to:

 Identify the different types of patterns in mathematics.


 Use inductive and deductive reasoning to predict patterns.
 Write expressions and statements in mathematical symbols.
 Explore on the different patterns found in objects, plants, and animals that
can be modelled mathematically.

LESSON 1:
NATURE AND CHARACTERISTICS OF
MATHEMATICS

What is Mathematics?

According to John William Navin Sullivan, a famous science writer,


“Mathematics, as much as music or any other art, is one of the means by which we
rise to a complete self-consciousness. The significance of mathematics resides

1
precisely in the fact that it is an art; by informing us of the nature of our own minds, it
informs us of much that depends on our minds.”

The word MATHEMATICS comes from the Greek μάθημά (máthēma), which,
in the ancient Greek language means “that which is learnt,” or “lesson” in Modern
Greek. Máthēma is derived from “manthano” while the modern Greek equivalent is
“mathaino” which means “to learn.” The study of mathematics and the use of
generalized mathematical theories and proofs are the key differences between Greek
mathematics and the mathematics of preceding civilization.

Hence, what is really mathematics? What is it exactly that mathematicians do


when they are doing mathematics?

Encyclopedia Britannica defines mathematics as the “science of structure,


order, and relations that has evolved from the elemental practices of counting,
measuring, and describing the shapes and characteristics of objects.” This definition
is one of the closest to mathematics that is evident in the modern world today.

Characteristics of Mathematics

What are the characteristics of mathematics? Among the characteristics of


mathematics are:

Classification

Logical
Generalization
Sequence

Characteristcs
Applicability Structure
of Mathematics

Mathematical
Precision and
language and
Accuracy
symbilism

Abstractness

2
Classification

Within logical-mathematical knowledge is the process of classification which


represents the first step towards learning more complex mathematical concepts.
Classification generates a series of mental relations through which objects are
grouped according to similarities and differences depending on specific criteria such
as shape, color, size, etc.

Logical Sequence

Ideas in mathematics need to flow in order that makes sense. The sequence can
naturally match what occurs in a text and what the reader needs to understand. It
means that each step can be derived logically from the preceding steps.

Structure

A structure on a set is an additional mathematical object that is related to that


given set in some particular characteristic or manner, and attaches (or relates) to that
set to endow it with some additional meaning or significance. The structure on a
particular mathematical set will allow mathematicians to study the set further and find
its relationship with objects.

Precision and Accuracy

Accuracy is how close a measured value is equal to the actual value. It is the
degree to which a given quantity is correct and free from error. Precision, on the
other hand, is how close the measured values are to each other.

Abstractness

It is the process of extracting the underlying essence of a mathematical concept


by taking away any dependence on real-world objects. Techniques and methods in
abstraction deal with quantitative relations and spatial forms and their
interrelationships with one another.

Symbolism

The language of mathematics is the system used by the mathematicians to


communicate mathematical ideas using symbols instead of words. This language is
uniquely constructed in such a way that all mathematicians understand symbiotic
notations and mathematical formulas.

Applicability

The applicability of mathematics can lie anywhere on a spectrum from the


completely simple to utterly complex. Mathematics can be used in all fields of human
endeavour. According to Kant, “in any special doctrine of nature, there can be only as
much proper science as there is mathematics therein.” Many agree that the problem
of understanding the significant link between mathematics and modern science is an
interesting and significant challenge for the philosopher of mathematics.

3
Generalizations

Making generalizations is fundamental to mathematics. It is a skill that must


be developed among students. It is of vital importance in a functioning society. The
Webster Dictionary defines generalization as the process of “finding and singling out
[of properties] in a whole class of similar objects.”

LESSON 2
MATHEMATICS AS A SCIENCE OF PATTERNS

“A mathematician, like a painter or a poet, is a master of patterns. If his


patterns are more permanent than theirs, it is because they are made with ideas.”

- Hardy (1992)

A pattern is a visible regularity in the world or in a man-made design. As


such, the elements of a pattern repeat in a predictable manner. Patterns, in a
mathematical sense, refer to the study of “tilings” “and wall-paper symmetries.” This
connotes order, regularity, and lawfulness. In the modern world, the patterns of
counting, measuring, reasoning, motion, shape, position, and prediction, reveal the
powerful influence mathematics has over people’s perception of reality.

Pattern recognition is a key determinant of logical, verbal, mathematical, and


spatial abilities. The most commonly used patterns today: the logic patterns, symbol
patterns, number patterns, and word patterns.

Logic Patterns

Logic patterns deal with the characteristics of various objects, order,


sequence and some possess similar attributes. These types of pattern are common
in aptitude tests. To construct or solve a pattern: 1) find the rule for the pattern, 2)
understand the nature of the sequence, and 3) analyze the difference between two
successive terms.

Example 1:

Answer: The lines rotate in a counter-


clockwise manner from the uppermost box
down to the fourth box. The fourth box in the
preceding object is the first box in the next
object. The pattern continues so the answer
is B.

4
Example 2:

Answer: The second figure is just the double of


the first figure, so it has two circles. The third
figure is the double of the second figure, thus it
has four circles. The fourth must be the double of
the third so it should have eight. Answer is D.

Example 3:

Answer: Look at the movements


of the arrows inside the triangles.
The arrow starts from a vertex
and the arrowhead points towards
the opposite side. So, the missing
figure should be B.

Example 4:

If the word MODERN can be encrypted as OGFGTP, how can you


code the word WORLD?

a. YQSNF c. YQUNF

b. YQTNF d. YQPNF

Answer: The correct answer is b. Each letter in the original word MODERN is
forwarded two places (+2) in the alphabetical order to get the encrypted word.

Example 5:

In a certain code language, the word PATTERNS can be written


TRTRTRTR, how would you write ALGEBRA in the same code language?

a. RGGRRRG c. GRGRGRG

b. GRRGGR d. RRGGRR

Answer: The correct answer is c. Each letter in the third, sixth, ninth, etc.
(multiples of 3) positions is reported according to the length of the given word.

5
Number Patterns

Working with number patterns leads directly to the concept of functions in


mathematics. The first step in determining the rule that defines the pattern is to look
for differences between the two consecutive numbers. The number pattern helps
make a generalization of how the numbers are arranged in a sequence. If there is no
logic (addition, subtraction, multiplication, division, squares, cubes, primes, etc) in the
differences, find other operations used in the pattern.

Try these examples; guess what the next number in the sequence.

Example 6:

11, 13, 17, 19, 23, ?

Answer: Notice that the differences between two consecutive numbers are 2,
4, 2, 4. However, this is not a pattern. Look upon the numbers again and
guess what? They are all consecutive prime numbers. So the next number
must be 29.

Example 7:

5, 7, 10, 15, 22, ?

Answer: The differences between each pair of consecutive terms in the


sequence are as follows: 2, 3, 5, 7. These primes are added to the previous
number to get the next number. 5+2=7, 7+3=10, 10+5=15, 15+7=22. So you
know that the next prime is 11. Hence, the next number in the pattern is
22+11=33.

Example 8:

5, 6, 10, 19, 35, ?

Answer: The differences between each number is 1, 4, 9, 16. These


numbers are squares of 1, 2, 3, and 4. So, the difference between 35 and the
next number should be the square of 5, which is 25. Thus, 35+25=60.

Word Patterns

Patterns can also be found in language. The metrical patterns of poems and
the syntactic patterns of how nouns are made plural or how verbs are changed to
past tense are both word patterns. Word patterns are used in decoding like:
consonant blends (words with a group of two or three consonants that each make
its own sound: grow, blend, sleeve, stair, sweet, free, blood, clothes), consonant
digraphs (words with two or three letters that come together to create a single
sound, e.g. chest, shop, sheep, brush, shirt, shade), vowel diphthongs (vowels that
glide in the middle: boil, now, soil, brown, cloud, rain, meat, bout, cow), and vowel

6
digraphs (a spelling pattern in which two or more adjoining letters represent a single
vowel sound: school, clean, each, feet, moon, cheese)

LESSON 3
THE FIBONACCI SEQUENCE

The Finonacci Sequence

Leonardo of Pisa, also known as


Fibonacci (1170 – 1250), is one of the best –
known mathematicians of medieval Europe. In
1202, after a trip that took him to several Aran and
Eastern countries, Fibonacci wrote the book Liber
Abaci. In this book Fibonacci explained why Hindu
– Arabic numeration system that he had learned
about the his travels was a more sophisticated
and efficient system than the Roman numeration Source: artifacts.com

system. This book also contains a problem created by Fibonacci that concerns the
birth of rabbits. Here is the statement of Fibonacci’s rabbit problem.

At the beginning of a month, you are given a pair of newborn rabbits. After a
month the rabbits have produced no offspring; however, every month thereafter the
pair of rabbits produces another pair of rabbits. The offspring reproduce in eaxctly the
same manner. If none of the rabbits dies, how many pairs of rabbits will be there be
at the start of each succedding month?

Source: eniscuola.net

7
The soliution of this problem is a sequence of numbers that we now call the
Fibonacci sequence.

The Fibonacci sequence is the series of numbers:

1, 1, 2, 3, 5, 8, 13, 21, 34….

The next number is found by adding up the two numbers before it:

The 2 is found by adding the two numbers before it (1+1),

The 3 is found by adding the two numbers before it (2+1),

And so on!

Remember!
The Fibonacci sequence can be written as a rule as:

X n=X n−1 + X n−2


where:
X n is term number “n” in the Fibonacci sequence

Example 9: Find the seventh and eighth Fibonacci numbers.

Solution:

For seventh Fibonacci number:

X n=X n−1 + X n−2

X 7 =X 7−1 + X 7 −2

X 7 =X 6 + X 5

X 7 =8+5

X 7 =13

For eighth Fibonacci number:

X n=X n−1 + X n−2

X 8=X 8−1 + X 8−2

X 8=X 7 + X 6

X 8=13+8

8
X 8=21
When we take any two successive Fibonacci numbers, their ratio is very close
to the Golden Ratio “φ” which is approximately 1.618034… In fact, the bigger the
pair of Fibonacci numbers, the closer the approximation. For example,

A B B/A

2 3 1.5

3 5 1.66666…

5 8 1.6

… … …

144 233 1.618055556…

233 377 1.618025751

And even more surprising is that we can calculate any Fibonacci number
using the Golden Ratio:

Remember!
Fibonacci number using the Golden Ratio:
n n
φ −( 1−φ )
x n= ∨xn =x n−1 × φ
√5

Example 10. Find the sixth Fibonacci number.

Solution:
n n
φ −( 1−φ )
x n=
√5
6 6
1.618034 … −( 1−1.618034 … )
x 6=
√5

x 6=8.00000033 ≈ 8

or

x n=x n−1 × φ

x 6=x 6−1 × 1.618034 …

x 6=5 ×1.618034 …

9
x 6=8.09017 ≈ 8

LESSON 4
MATHEMATICS FOR OUR WORLD

Millions of patterns can be found in the environment. These patterns occur in


various forms and in different contexts which can be modelled mathematically. Some
examples are symmetries, trees, spirals, waves, tessellations, stripes, meanders,
cracks, and many more.

The German biologist Ernst Haeckel painted marine organisms to show their
symmetry. Scottish biologist D’Archy Thompson studied the growth patterns in plants
and animals. Beehives are one of the examples of patterns in nature that can be
modelled mathematically. They are made of walls, each of the same size, enclosing
small hexagonal cells where honey and pollen are stored and bees are raised.
Honeycombs are examples of wallpaper symmetry.

Scientists theorize that sunflowers can pack in the maximum number of seeds
if each seed is separated by an irrational-numbered angle. In mathematics, the most
irrational-number is known as the Golden Ratio, or Phi. It is a mathematical ratio
that’s commonly found in nature. It can be used to create visually-pleasing, organic-
looking compositions in your design projects or artwork.

Here are some examples of patterns in nature.

Source: curiodyssey.org Source: blog.csiro.au

Honeycomb Animal Skin

Source: kidsdiscover.com
Source: viewbug.com
10
Plant Trees
Source: panachallenge.com Source: thescienceexplorer.com
Fruit Flower

Source: pinterest.com
Sand
Source: iphone6papers.com

Water

It could be noted that mathematics exists in the natural world. It is specifically


on how nature behaves and the patterns it follows through logical and mathematical
structures. It is important that this concepts and ideas be used to further advance
what we know and provides substantial and effective solutions to some underlying
problems we faced nowadays.

Mathematics is an important tool in generating models, analysis and


interpretations that could be used in predicting the possible outcomes of a certain
event. Like for examples, to predict the probability of a storm to occur, the possible
return period of an event having the same magnitude, determine the trend of social
media users, predict the increase of the world’s population after 10 years, and others.
This may give a clearer picture to innovators, scientists, engineers and other
professionals, even common people, on the possible scenarios that may happen.
This may lead to creating strategies to lessen the impact and several interventions to
addressed the possible outcomes.

In recent years, global warming and climate change, are hot topics that leads
to several debates, protests and legislations. Scientists warns that if man will not
change his behavior towards the environment, the effects of climate change is

11
inevitable. Through the use of mathematics, man is able to exert control over himself
and possibly reverses the effects of climate change on nature.

We have identified some of the applications of mathematics and that it plays


a bigger role in the societal context. We know that mathematics is with us in
everything we do. Although we may not notice it. Knowledge of mathematics is
helpful in every endeavor. So, it is better to embrace it to have a better understanding
of the world we live in and be part of the crusade in protecting and conserving our
one and only natural world.

Chapter
Test I: Exercises
Write the letter of the correct answer. Justify your answer.

1. What figure should come next in this sequence?

2. Which figure is next in the sequence?

3. Which figure can be used to continue the series?

12
4. What figure should come next in this sequence?

5. Which figure is next in the sequence?

Test II. Identify the next number in the pattern.

1. 100, 98, 96, 94, ___

2. 12, 13, 15, 18, 22, ___

3. 7, 20, 47, 94, 167, ___

4. 1, 4, 2, 8, 6, 24, 22, ___

5. 11, 23, 48, 99, ___

6. 6, 13, 27, 55, 111, ___

7. 8, -24, 72, -216, 648, -1994, ___

8. 5, 27, 137, 687, 3437, ___

9. 34, 7, 37, 14, 40, 28, ___

10. 16384, 8192, 2048, 256, 16, ___

TEST III. Find the nth term of the Fibonacci sequence. Round off to whole numbers
(φ = 1.618034)

13
1. X 6 3. X 33

2. X 9 4. X 12

5. X 25 8. X 23

6. X 11 9. X 27

7. x20 10. x18

TEST IV. Find the ratio of the two consecutive Fibonacci numbers. . Round off to 7
decimal places

1. X 6 and X 7

2. X 10 and X 11

3. X 17 and X 18

4. X 4 and X 5

5. X 12 and X 13

Test V. Reflection Paper

Direction: Explore and take photos of different patterns found in objects, plants, and
animals are present in your surroundings that can be modelled
mathematically. After such, make a reflection paper on the nature of
mathematics. You may include your photos in your reflection paper. You
will be graded with your reflection paper using the rubrics below.

Category Description Point Value


Mechanics Well-written and well-
2
Well written, clear organization, organized
uses standard English grammar, Inadequate due to lack of
contains minor, if any, spelling organization, grammar,
1
errors and/or major spelling
errors
Mechanics – Highest Possible Score 2
Evidence of Critical Thinking Demonstrates critical
Critical thinking includes thinking and the ability to 3
application, analysis, synthesis apply concepts.
and evaluation. Arguments are Demonstrates some critical
clear and show depth of insight thinking and application of 2
into theoretical issues, originality concepts
of treatment, and relevance. Shows minor or incorrect 1

14
May include unusual insights. application of concepts
Arguments are well supported.
Evidence of Critical Thinking– Highest Possible Score 3
Development of Ideas Well developed; shows
evidence of reflection
Well-developed thoughts, ideas, and/or metacognition; new
3
and details, which shows ideas introduced and
evidence of reflection, new reflects a good grasp of
ideas, and grasp of concepts. concepts presented.
Shows some evidence of
reflection, but not well-
developed; few new ideas
2
introduced but reflects a
grasp of concepts
presented.
Not much thought or detail;
shows little evidence of
reflection or grasp of 1
concepts; no new ideas
introduced.
Development of Ideas– Highest Possible Score 3
Total Possible Score 8

Source: Grading Rubrics for Reflection Journal Entries retrieved from


https://people.richland.edu/fbrenner/syllabus/reflectrubric.html

15
Chapter 2:
Speaking Mathematically

Introduction
M
athematics can be thought of as a language filled with vocabulary,
symbols, and sentence structures. Learning both mathematics and the language of
mathematics is a challenge for all. These can make things difficult for all who wish to
relate math to their everyday language and experiences. And like any language, we
have to speak math proficiently in order to use it efficiently. With these, this chapter is
designed to introduce you to the language of sets and relations and functions. As you
work through this module, you will be introduced to different notations and concepts
on sets, relations, and functions that will help building and deepen your knowledge in
learning mathematics.

Learning Outcomes:
At the end of this chapter, you are expected to:

 Describe sets.
 Solve problems involving operation of sets.
 Defferentiate function and relation.
 Recognize relation that are not a function.
 Find the domain and range of relation.

LESSON 1:
THE LANGUAGE OF SETS

Set

The concept of set is basic and it is impossible for us to define what set is.
Set is simply described in terms of the properties of those objects that belong to the
set. Hence, set is described as any group or collection of objects whose properties
are well defined. This means two things: the objects in the set, called elements must
be distinguishable from each other and description of the elements must be clear and
unambiguous.

16
Set can be identified or named with the use of any capital letter or symbols.
Elements of sets are written with small letters or even the name of object itself,
separated by commas, within its braces. The symbol ∈ is utilized to indicate the
membership of an obejct in a set (element) and the symbol for not being a member
or does not belong in a set (not an element).

Example 1: Determine whether each statement is set or not. If set, list down the
elements.

1. The set of all positive numbers less than 8.


2. The set of months with 30 days.
3. The set of even numbers divisible by 3 and less than 50.
4. The set of tall people.
5. The set of nice cars.

Solution:

1. The statement is a set.


N = {1, 2, 3, 4, 5, 6, 7}

2. The statement is a set.


M = { September, November, April, June}

3. The statement is a set.


S = {6, 12, 18, 24, 30, 36, 42, 48}

4. The statement is not a set because people tend to disagree what “tall”
means. The word “tall” is not precise.
5. Same with number 4, the statement is not a set because people would not
have the same perception of what “nice” car is. The word “nice” is not
precise.

Example 2: Determine whether each statement is true or false.

1. 6 is an element of all positive numbers less than 8.


2. 8 ∈ { 4, 5, 6, 7}
3. –6∈N
4. ½ Z

Solution:

1. Since 6 is an element of positive numbers less than 8, then the statement


is true.
2. 8 is not an element of the give set, so the statement is false.
3. There is no negative natural number or counting number, so the
statement is false.
4. Since ½ is not an element of integer, then the statement is true.

17
Basic Number Sets

The following sets of numbers are commonly used in many areas of


mathematics.

Remember!
Natural Number or Counting Numbers N = { 1, 2, 3, 4, 5, ...}
Whole Numbers W = { 0, 1, 2, 3, 4, 5, ...}
Integers Z = { ..., -4, -3, -2, -1, 0,1, 2, 3, 4, 5, ...}
Rational Numbers Q = the set of all terminating or repeating decimals
Irrational Numbers Q’ = the set of all nonterminating, nonrepeating decimals
Real Numbers R = the set of all rational and irrational numbers

Set Description

In mathematics several ways the elements of a set can be described but for
simplicity of our discussion, we will be using the two methods: Roster and Rule
Method.

1. Roster Method. Indicate a set by listing the elements separated by


commas and enclosing them in braces.

2. Rule Method. Describing the the set using words and enclosing them in
braces.

Example 3: Use the roster and rule methods to represent the following sets.

1. The set of the days in a week.


2. The set of numbers divisible by 5 less than 50

Solution:

1. Roster Method: A = { Monday, Tuesday, Wednesday, Thursday, Friday,


Saturday, Sunday}
Rule Method: A = {x | x is a day in a week}

2. Roster Method: B = { 5, 10, 15, 20,25, 30, 35, 40, 45}


Rule Method: B = { x | x is a number divisible by 5 less than 50}

Note: The vertical bar is read “such that” and the statement 2 is read as,
B is the set of all x such that x is a number divisible by 5 less than 50.

18
Types of Sets

1. Finite set. A set is said to be a finite set if all possible elements can be
enumerated or listed down.

Example 4: The set of vowel letters in the English alphabet.


A = {a, e, i, o, u}

2. Infinite set. A set is said to be an infinite set if all possible elements are
uncountable or unlimited and cannot be listed down. Since not all elements
can be enumerated, ellipses (...) is used to indicate the continuity of the
elements.
Example 5: The set of counting numbers greater than 20.
A = {21, 22, 23, 24, 25, 26, 27, 28, 29, 30, 31, ...}

3. Empty set. Also known as null set. A set that contains no elements. The
symbol Ø or { } is used to represent the empty set.

Example 6: The set of counting numbers between 5 and 6.


A=Ø

4. Unit set. A set that contain only one element.

Example 7: A set of counting number less than 2.


A = {1}

5. Equivalent sets. Two sets A and B are said to be equivalent , denoted by A


~ B, if and only if A and B have the same number of elements.

Example 8: A = { 1 , 2, 3, 4, 5} and B = {a , b, c, d, e}. Since sets A and b


have the same number of elemens, then A ~ B.

6. Equal sets. Two sets A and B are said to be equal, denoted by A = B, if and
only if A and B have exactly the same elements.

Example 9: A = {l, o, v, e} and B = {e, l, o, v}. Since sets A and b have the
same elemens, then A = B.

7. Joint sets. Two sets A and B are said to be joint set if they contain at least
one element in common.

Example 10: If A = {1, 4, 5, 6} and B = { 2, 4, 8, 9}, the the sets are joints
sets since they contain one common element, 4.

19
8. Disjoint sets. Two sets A and B are said to be disjoint set if they do not
contain common element.

Example 11: A = { 1, 2, 3, 7} and B = {4, 5, 9}. Set A and set B are disjoint
since there is no common element.
9. Subset. If A and B are two sets, and every element of set A is also an
element of set B, then A is called a subset of B and we write it as A ⊆ B.

Example 12. Let A = {2, 4, 8} and B = {2, 4, 6, 8, 10}. Since, all the elements
of set A are contained in set B, then A ⊆ B.

10. Super set. Whenever a set A is a subset of set B, we say that B is a superset
of A and we write, B ⊇ A.

Example 13. Let A = {2, 4, 8} and B = {2, 4, 6, 8, 10}. Here, A ⊆ B but B ⊇ A.

11. Proper subset. If A and B are two sets, then A is called the proper subset of
B, denoted by A ⊂ B, if A ⊆ B and B ⊇ A but A ≠ B.

Example 14. If A = {1, 2, 3, 4} and B = {1, 2, 3, 4, 5}, we observe that, all the
elements of A are present in B but the element ‘5’ of B is not
present in A. So, we say that A is a proper subset of B.
Symbolically, we write it as A ⊂ B.

12. Power set. The collection of all subsets of set A is called the power set of A.
It is denoted by P(A). In P(A), every element is a set including an empty set of
A. The number of subsets can be determined by using the formula 2 n, where
n refers to the number of elements in a given set.

Example 15: If A = {a, b, c} then the number of subsets of A is


23 = 8. Then all subsets of a will be
P(A) = {Ø,{a}, {b}, {c}, {a, b}, {a, c}, {b, c}, {a, b, c}}

13. Universal Set. If the set has all the possible elements under consideration,
then the set is know as universal set. It is denoted by U.

Example 16. If A = {1, 2, 3}, B = { 3, 4, 5} and C = { 6, 7, 8}, then the


U = {1 , 2, 3, 4, 5, 6, 7, 8}

Operation of Sets

Set can be combined in a number of different ways to produce another set.


There are operations that involved in a set. The following operations are union,
intersection, complementation, set difference, and cartesian product.

1. Union of Two Sets

20
The symbol U is employed to denote the union of two sets. Let us
have two sets A and B in symbol A ∪ B read “A union B” or “the union of set
A and Set B” – is defined as the set that consist of all elements belonging to
either set A or set B (or both).

A U B = {x | x ∈ A or x ∈ B}

Example 17: Find the union of the following sets:


1. A = {2, 3, 4} and B = { 3, 4, 5}
2. D = {1, 2, 3, 4, 5, 6, 7, 8, 9} and E = {5, 10, 15}

Solution:
1. A U B = {2, 3, 4, 5}
2. D U E = { 1, 2, 3, 4, 5, 6, 7, 8, 9, 10, 15}

Note: Elements of the union that are elements of both sets are listed only
once.

2. Intersection of two Sets

The intersection of the sets A and B, denoted by A ∩ B, read “A


intersection B” or “the intersection of set A and set B” is defined as the set of
elements belongs to both A and B i.e. set of the common element in A and B.

A Ո B = { x | x ∈A and x ∈ B}

Example 18: Find the intersection of the following sets:


1. A = {1, 2, 3} and B = {1, 2, 4, 5}
2. C = {1, 3, 5} and D = {2, 4, 6}

Solution:
1. A Ո B = {1, 2}
2. C Ո D = Ø

Note: What happened in example 2? Since there are no common


elements to both two sets, then the intersection of the sets would be a
null or empty set.

3. Complement of a set

Let us consider set A. The complement of set A denoted by A’ read as


“A prime” is defined as the set whose elements are not in set A but elements
of the universal set U. in symbols,

A' = { x | x A but x ∈ U}

21
Example 19: Find the complement of set A = {2, 4, 6, 8} and U = {1, 2, 3, 4, 5,
6, 7, 8, 9, 10}

Solution:
A’ = {1, 3, 5, 7, 9, 10}

Example 20: Let us consider the following sets:


U = {1, 2, 3, 4, 5, 6, 7, 8, 9, 10}
A = {1, 3, 5, 7, 9}
B = {2, 4, 5, 8, 9}

Give the elements of:


a. A’
b. (A Ո B)´

Solution:
a. A’ = {2, 4, 6, 8, 10}
b. (A Ո B)´ = (A Ո B) = {5, 9}
(A Ո B)´ = {1, 2, 3, 4, 6, 7, 8, 10}
4. Set difference

Given set A and set B, the set difference of set B from set A is the set
of all elements in A, but not in B. In symbols:

A - B = { x | x ∈ A but x B}

Example 21: Find the difference.

1. Given set A = {a, b, c, d, e, f, g} and B = {a, c, f, h, k, u}, find A−B .


2. Find B− A from the given sets below.
A = {x | x is a number greater than 6 and less than 10}
B = { x | x is a number less than 15}

Solution:
1. A – B = { b, d, e, g}
2. First write the sets in roster method to view the elements
A = { 7, 8, 9}
B = { 1, 2, 3, 4, 5, 6, 7, 8, 9, 10, 11, 12, 13, 14}
B – A = {1, 2, 3, 4, 5, 6, 10, 11, 12, 13, 14}

5. Cartesian Product

The cartesian product of sets A and B, denoted A × B, is the set of all


possible ordered pairs where the elements of A are first and the elements
of B are second.

In set-builder notation, A × B = {(a, b) : a ∈ A and b ∈ B}.

22
Example 22. Find A x B, whose set A={ H ,T } and B= {1 , 2 ,3 , 4 , }

Solution:

A x B= { ( H ,1 ) , ( H ,2 ) , ( H ,3 ) , ( H , 4 ) , ( T ,1 ) , ( T , 2 ) , (T , 3 ) , ( T , 4 ) }

Example 23. A couple is planning their wedding. They have three nieces
(Ann, Betty, and Deanne) and two nephews (Ed, Fred). How
many different pairings are possible to have one boy and one girl
as a ring bearer and flower girl?

Solution:
Note that this problem can be considered as cartesian product

Let A be the set of nieces A = {Ann, Betty, Deanne}


Let B be the set of nephews B = {Ed, Fred}
A x B = { (Ann, Ed), (Ann, Fred), (Betty, Ed), (Betty, Fred), (Deanne,
Ed), (Deanne, Fred)}

Thus, the number of pairs is 6.

Venn Diagram

A Venn Diagram is a pictorial representation of the relationships between


sets. We can represent sets using Venn diagrams. In a Venn diagram, the sets are
represented by shapes; usually circles or ovals. The elements of a set are labeled
within the circle.
The following diagrams show some set operations and Venn diagrams for
complement of a set, disjoint sets, subsets, intersection and union of sets.

Set A A’ complement of A

A and B are disjoint sets B is a proper subset of A

U 23 U
The set of all elements being considered, the universal set is represented by
a rectangle.

Example 24. In a class of 80 students, each of the students passed either in


Mathematics or in English or in both. 50 students passed in both and
20 passed in English. Find how many students passed in
Mathematics?

Solution: Let "x" be the number of students passed in Mathematics. By representing


the given details in Venn diagram, we get

x 50 20

M E

From the Venn diagram:

x + 50 + 20 = 80
x = 80 - 70 = 10
x = 10

Number of students passed in Mathematics

= x + 50 = 10 + 50 = 60

Therefore, 60 students passed in Mathematics.

Example 25. The population of a town is 5000. Out of these, 1500 persons read
newspaper A and 2700 read newspaper B. 1000 persons read both
the newspapers. Find the number of persons who do not read either of
the two papers.

Solution: Let: A = The set of persons who read newspaper A


B = The set of persons who read newspaper B

1500 2700
-1000 1000 -1000
500 170
A 0 B

24
Number of persons who read at least one newspaper

= 500 + 1000 + 1700


= 3200
To find the number of persons who do not read either of the two papers,
we have to subtract number of persons who read at least one from total
population.

= 5000 - 3200
= 1800

So, the number of persons who do not read either of the two papers is
1800.

Example 26. In a school, all the students play either basketball or volleyball or both.
80 students play basketball, 75 students play volleyball and 60 students
play both games. Find
a. the number of students who play Basketball only.
b. the number of students who play Volleyball only.
c. the total number of students in the school.

Solution: Let B = The set of students who play basketball


V = The set of students who play volleyball

80 75
- 60 60 - 20
20 55
B V

a. The number of students who play basketball only is 20

b. The number of students who play volleyball only is 55

c. The total number of students in the school


= 20 + 55 + 60
= 135

Example 27. There are 100 students surveyed and asked which of the
following subjects they take this semester: Mathematics, English, or
Biology. The result revealed that 35 responded English, 50
responded Mathematics, 29 responded Biology, 12 responded
Mathematics and English, 8 responded English and Biology, 11
responded Biology and Math, and 5 responded all. Determine

a. How many students are not taking any of the three


subjects?

25
b. How many students take Math, but not Biology or English?
c. How many students take Math and English, but not
Biology?

Solution: In this problem, we have four regions which overlap. The easiest strategy
is to start at the center, the part where the three circles overlap. In short,
we start from bottom to the top in the result above.

1. Five students responded that they took all


the subjects, so we put 5 at the center.

2. Eleven responded Biology and Math. So,


we should put it in the Biology-Math
overlap. However, of the 11 who takes
Biology and Math, 5 were also
taking English as shown in (1). So, there
are 11-5 = 6 students in the Math-
Biology overlap.

3. Eight responded Biology and English. But of


those 8 taking Biology and English, 5 are also
taking Math. So, there are 8 – 5 = 3 students
who are taking Biology and English.

Also, there are 12 students who are taking


Math and English, and of those 12, 5 are
also taking Biology, so there are 12
– 5 = 7 who are taking Math and English.
Now see the next figure to see how the
Venn diagram should look like after this
step.
4. Next, 29 students responded Biology. But
notice that 6, 5, and 3 are already in the
Biology circle. So, we subtract those students
from 29. That is, 29 – (6 + 5 + 3) = 29 – 14 =
15. So, there are 15 students who take only
Biology.

5. Lastly, there are 50 students who are taking


Math and 35 who are taking English. But in
the Math circle there are 6 + 5 + 7 students
who are also taking the other subjects and in
the English circle there
are 3 + 5 + 7 students who are also taking
the other subjects. Therefore, we can have
the following calculations: 26

Number of students who take only Math =


50 – (6 + 5 + 7) = 32
a. To answer the first question, how many students did not take any of the three
subjects, recall that there are 100 students who were surveyed. If we add all
the numbers in the diagram, 15 + 32 + 20 + 6 + 7 + 3 + 5, the sum is only 88.
Therefore, 100 – 88 = 12 students did not take any of the three subjects.

b. The number of students taking Math, but not Biology or English (another way
to rephrase it, is how many students take only Math) is 32.

c. The number of students who are taking Math and English is 7.

LESSON 2
THE LANGUAGE OF RELATIONS AND FUNCTIONS

Relation

When you group two or more points in a set, it is referred to as a relation.


When you want to show that a set of points is a relation, you list the points in order
pairs. A relation is just a set of ordered pairs. There is absolutely nothing special at
all about the numbers that are in relation. In other words, any bunch of numbers is a
relation so long as these numbers comes in pair.

Example 28: {(2, 4), (3, 5), (4, 8), (6, 7) usually written in set notation.

where: Range or inputs (x – values) = {2, 3, 4, 6}


Domain or outputs (y – values) = {4, 5, 8, 7}

27
You will notice in the above example that, relation shows the relationship
between INPUT and OUTPUT, merely the range and domain. There are other ways
to write the relation aside from set notation such through tables, plotting in XY
coordinates and mapping diagram as shown below;

X Y y

2 1 2 1

2 -2 2 -2
x
0 -3 -3
0
1 4
1 4
3 1
3

Relation in table Relation in Graph Relation in mapping diagram

What is a Function?

Function relate an input to an output


Range or (independent
Input
variable)
y=f ( x )

output
Name of function
Domain or (dependent
variable)

A function is a relation when each element of the domain is paired to exactly


one element in the range. In other words, there is one and only one output (y) with
each input (x).

Function can be illustrated as of the following;

1. Set of Order pairs

{ ( 3 , 5 ) , ( 6 , 8 ) , ( 2 , 4 ) , (7 ,1 ) } Function

{ ( 4 , 2 ) , ( 3 , 2 ) , (5 ,2 ) , ( 6 , 2 ) } Function

28
{ ( 1 , 2 ) , ( 0 , 3 ) , ( 1 ,6 ) , ( 5 , 4 ) } Not a Function

The first two examples are functions since for every one output there
is one corresponding input unlike the other two examples that there is two or
more output to a single input, therefore it not a function. Note that, the first
element (x – Domain) or the independent variable, while the second element
(y – range) or the dependent variable.

2. Mapping or arrow diagram

Examples:

3 a 3
4 b 4
5 5 8
c 9
Function One to One Relation Function Many to One Relation

3 3 1
4 4 7
5 9
5

Not function, One to Many Relation Not function, not all elements are represented

3. Graphing

Vertical line test

y y

x x

Function Function Function

Vertical line passes only once in the graph; therefore it is a function

29
Not Function Not Function

Vertical line passes through more than one point of the graph, then that
relation is not a function.

Evaluating a Function

When we have a function in formula form, it is usually a simple matter to


evaluate the function. For example, the function f ( x )=5−3 x 2 can be evaluated by
squaring the input value, multiplying by 3, and then subtracting the product from 5.

How will you evaluate a function given its formula?

Follow this simple step:

1. Replace the input variable in the formula with the value provided.
2. Calculate the result.

Example 29: Given the function f ( x )=x 2 +2 x when x = 4

Solution:

2
f ( 4 )=( 4 ) + 2 ( 4 ) replacing the input in the formula

f ( 4 )=16+8 calculating the result

f ( 4 )=24 answer

30
Chapter
Exercises
Direction: Answer the following. Show all pertinent solutions.

1. If A = {1, 3, 5}, B = {3, 5, 6} and C = {1, 3, 7}

a. Verify that A ∪ (B ∩ C) = (A ∪ B) ∩ (A ∪ C)
b. Verify A ∩ (B ∪ C) = (A ∩ B) ∪ (A ∩ C)

2. Let A = {a, b, d, e}, B = {b, c, e, f} and C = {d, e, f, g}

a. Verify A ∩ (B ∪ C) = (A ∩ B) ∪ (A ∩ C)
b. Verify A ∪ (B ∩ C) = (A ∪ B) ∩ (A ∪ C)

3. If A = {1, 3, 5} and B = {2, 3}, then

a. Find A x B
b. BxA
c. AxA
d. BxB
4. Let U ={ 1, 2 , 3 , 4 , 5 , 6 ,7 ,8 , 9 , 10 }
T ={ 2 , 4 , 5 , 6 , 8 , 9 }
J= {1 , 3 , 5 ,7 , 9 , 10 }

Find the elements of:


a. ( J ∩T ) x J
'
b. J ∩T '

5. A survey of faculty and graduate students at the University of Florida's film


school revealed the following information: 51 admire Moe, 49 admire Larry,
60 admire Curly, 34 admire Moe and Larry, 32 admire Larry and Curly, 36
admire Moe and Curly, 24 admire all three of the Stooges, and 1 admires
none of the Three Stooges.
a. How many people were surveyed?
b. How many admire Curly, but not Larry nor Moe?
c. How many admire Larry or Curly?
d. How many admire exactly one of the Stooges?
e. How many admire exactly two of the Stooges?

31
6. An advertising agency finds that, of its 170 clients, 115 use Television, 110
use Radio and 130 use Magazines. Also 85 use Television and Magazines,
75 use Television and Radio, 95 use Radio and Magazines, 70 use all the
three. Draw Venn diagram to represent these data. Find

a. how many uses only Radio?


b. how many uses only Television?
c. how many use Television and Magazine but not radio?

7. What is the domain of the following ordered pairs? List your answer in an
increasing order.{ ( 12 , 5 ) , ( 4 , 10 ) , ( 2, 6 ) , ( 7 , 9 ) }

8. What is the range of the following ordered pairs? List your answer in an
increasing order: { ( 4 ,−3 ) , ( 12 ,−14 ) , (11 ,1 ) , (−2 , 4 ) , ( 9 , 7 ) }

9. Which of this could be a function? Why?

a) { ( 1 , 4 ) , ( 3 , 2 ) , (2 , 7 ) , ( 0 , 6 ) }
b) { ( 2 , 8 ) , (−2 ,9 ) , (−2 , 10 ) , (−3 ,11 ) }
c) { ( 3 , 5 ) , ( 3 ,2 ) , (−3 , 4 ) , ( 3 , 3 ) }
d) { ( 1 , 3 ) , (3 , 8 ) , ( 3 , 1 ) , (−2 , 2 ) }

10. Which of these mapping diagrams represent a function? Why?

a. b.

11 -2
3 9
-3 0
-1 12
1 14
8 15
5 -6

11. Which of these is a function? Why?

a. b.

32
12. Fill in the table base using the equation y=−2 x

X 1 -1 -2 2

13. Evaluate f ( x )=−3 x 2 + x +7 when x = 2.


14. Evaluate f ( x )=12 x 2+ x−8 when x = 2
15. People often buy different things from the supermarket. If one constructs a
table with the cost of the supermarket shopping and the time spent on
shopping as variables, will the ordered pairs from this data represent a
function or a relation? Why?

33
Chapter 3:
Problem Solving

Introduction
M
ost occupations require good problem-solving skills. For instance, architects
and engineers must solve many complicated problems as they design and construct
modern buildings that are aesthetically pleasing, functional, and that meet stringent
safety requirements. Two goals of this chapter are to help you become a better
problem solver and to demonstrate that problem solving can be an enjoyable
experience.

Learning Outcomes:
At the end of this chapter, you are expected to:

 Differentiate inductive reasoning from deductive reasoning.


 Use different types of reasoning to justify statements and arguments made
about mathematics and mathematical concepts.
 Solve problems using inductive and deductive reasoning.
 Solve problems involving patterns and recreational problems.
 Solve problems employing Polya’s four steps.
 Increase awareness on the importance of reasoning and problem solving
 Organize their methods and approaches for providing and solving
problems

LESSON 1:
INDUCTIVE AND DEDUCTIVE REASONING

Inductive Reasoning

Inductive and deductive reasoning are two fundamental forms of reasoning


for mathematicians. Inductive reasoning involves looking for patterns and making

34
generalizations. For example, students use this type of reasoning when they look at
many different quadrilaterals, and try to list the characteristics they have in common
(see Figure 3a). The reasoning process is enhanced by considering figures that are
not quadrilaterals and discussing how they are different.
An example of inductive reasoning is, for example, when you notice that all
the dogs you see around are black and white so you make the conclusion that all
dogs in the world are black and white. Can you say for certain that this conclusion is
correct? No, because it is based on just a few observations. However, it is the
beginning of forming a correct conclusion, or a correct proof. What this observation
has given you is a starting hypothesis to test out.

Remember!
Inductive reasoning is the process of reaching a general conclusion
by examining specific examples.

When you examine a list of numbers and predict the next number in the list
according to some pattern you have observed, you are using inductive reasoning.

Example 1. Use Inductive Reasoning to Predict a Number

Use inductive reasoning to predict the next number in each of the


following lists.

a. 3, 6, 9,12, 15, _____ b. 1, 3, 6, 10, 15, _____

Solution:

a. Each successive number is 3 greater than the preceding number. Thus,


we predict that the next number in the list is 3 greater than 15, which is
18.
b. The first two numbers differ by 2. The second and the third numbers differ
by 3. It appears that the difference between any two numbers is always 1
more than the preceding difference. Since 10 and 15 differ by 5, we
predict that the next number in the list will be 6 larger than 15, which is 21.

Inductive reasoning is not used just to predict the next number in a list. In
Example 2, we use inductive reasoning to make a conjecture about an arithmetic
procedure.

Example 2. Use Inductive Reasoning to Make a Conjecture

Consider the following procedure: Pick a number. Multiply the number


by 8, add 6 to the product, divide the sum by 2, and subtract 3.
Complete the above procedure for several different numbers. Use
inductive reasoning to make a conjecture about the relationship

35
between the size of the resulting number and the size of the original
number.

Solution:
Suppose we pick 5 as our original number. Then the original number
would produce the following results:
Original number: 5
Multiply by 8: 8 x 5 = 40
Add 6: 40 + 6 = 46
Divide by 2: 46 ÷ 2 = 23
Subtract 3: 23 – 3 = 20
We started with 5 and followed the procedure to produce 20. Starting with 6
as our original number produces a final result of 24. Starting with 10 produces a final
result of 40. Starting with 100 produces a final result of 400. In each of these cases,
the resulting number is four times the original number. We conjecture that following
the given procedure produces a number that is four times the original number.

Counterexamples

A statement is a true statement provided that it is true in all cases. If you can
find one case for which a statement is not true, called a counterexample, then the
statement is a false statement. In Example 3, we verify that each statement is a false
statement by finding a counterexample for each.

Example 3. Find a Counterexample

Verify that each of the following statements is a false statement by


finding a counterexample.

For all numbers x:

a. |x| > 0 b. x2 > x

Solution:

A statement may have many counterexamples, but we need only to find one
counterexample to verify that the statement is false.

a. Let x = 0. Then /0/ = 0. Because 0 is not greater than 0, we have found


a counterexample. Thus, “for all numbers x, /x/ > 0” is a false statement.
b. For x = 1, we have 12 = 1. Since 1 is not greater than 1, we have found
a counterexample. Thus, “for all numbers x, x2 > x” is a false statement.

QUESTION: How many counterexamples are needed to prove that a statement is


false?

36
Deductive Reasoning

Another type of reasoning is called deductive reasoning. Deductive


reasoning is distinguished from inductive reasoning that it is the process if reaching
a conclusion by applying general principles and procedures. Deductive reasoning
involves making a logical argument, drawing conclusions, and applying
generalizations to specific situations. For example, once students have developed an
understanding of “triangle”, they apply that generalization of new figures to decide
whether or not each is a triangle. The conclusions reached by this type of reasoning
are valid and can be relied on.

Remember!
Deductive reasoning is the process of reaching a conclusion by
applying general assumptions, procedures or principles.

Illustrative Examples
1. If a number is divisible by 2, then it must be even.
12 is divisible by 2.
Therefore, 12 is an even number.

2. All Mathematics teachers know how to play Sudoku.


Resty is a Math teacher.
Therefore, Resty knows how to play Sudoku.

3. If a student is a DOST scholar, he receives a monthly allowance.


If a student receives a monthly scholar, his parents will be happy.
Therefore, if a student is a DOST scholar, his parents will be happy.

4. If ∠A and ∠ B are supplementary angles, their sum is 180°.


If m⦞∠A = 100°, then m∠ B = 80°.

Example 4. Use Deductive Reasoning to Make a Conjecture

Use deductive reasoning to show that the following procedure produces


a number that is four times the originall number.

Procedure: Pick a number. Multiple the number by 8, add 6 to the


product, divide the sum y 2, and subtract 3.

Solution:
Let n represents the original number.

37
Multiply the number by 8: 8n
Add 6 to the product: 8n + 6
Divide the sum by 2: (8n + 6) / 2 = 4n + 3
Subtract 3: 4n + 3 – 3 = 4n

Inductive Reasoning vs Deductive Reasoning

In example 5, we analyze arguments to determine whether they use inductive


reasoning or deductive reasoning.

Example 5. Determine Types of Reasoning

Determine whether each of the following arguments is an example of


inductive reasoning or deductive reasoning.

a. During the past 10 years, a tree has produced plums every other
year. Last year, the tree did not produce plums, so this year the tree
will produce plums
b. All home improvement cost more than the estimate. The contractor
estimated that my home improvement will cost P70,000. Thus, my
home improvement will cost more than P70,000.

Solution:

a. This argument reaches a conclusion based on specific examples, so it is


an example of inductive reasoning.
b. Because the conclusion is a specific case of a general assumption, this
argument is an example of deductive reasoning.

Logic Puzzles

Logic puzzles can be solved by using deductive reasoning and a chart that
enables us to display the given information in visual manner.

Example 6. Solve a Logic Puzzle

Each of four neighbors, Mark, Zen, Linda, and Roy, has a different
occupation (teacher, banker, chef, or broker). From the following clues,
determine the occupation of each neighbor.

1. Zen gets home from work after the banker but before the broker.
2. Linda, who is the last to get home from work, is not the teacher.
3. The broker and Linda leave for work at the same time.
4. The banker lives next door to Roy.

38
Solution:

From clue 1, Zen is neither the banker nor the broker. In the following
chart, write X1 (which stands for “ruled out by clue 1”) in the Banker and the
Broker columns of Mark’s row

Teacher Banker Chef Broker


Mark
Zen X1 X1
Linda
Roy

From clue 2, Linda is not the teacher. Write X2 (ruled out by clue 2) in
the teacher column of Lind’s row. We know from clue 1 that the banker is not
the last to get home, and we know from clue 2 that Linda is the last to get
home; therefore, Zen is not the banker. Write X2 in the Banker column of
Linda’s row.

Teacher Banker Chef Broker


Mark
Zen X1 X1
Linda X2 X2
Roy

From clue 3, Linda is not the dentist. Write X3 for this condition. There
are now Xs for three of the four occupations in Linda’s row; therefore Linda
must be the Chef. Place a / (check mark) in that box. Since Linda is the chef,
none of the other three people can be the chef. Write X3 for these conditions.
There are now Xs for three of the four occupations in Zen’s row; therefore,
Zen must be the teacher. Insert a / (check mark) to indicate that Zen is the
teacher, and write X3 twice to indicate that neither Mark nor Roy is the
teacher.

Teacher Banker Chef Broker


Mark X3 X3
Zen / X1 X3 X1
Linda X2 X2 / X3
Roy X3 X3

From clue 4, Roy is not the banker. Write X4 for this condition. See
the following table. Since there are three Xs in the Banker column, Mark must
be the Banker. Place a / in that box. Thus Mark cannot be the broker. Write
X4 in that box. Since there are Xs in the broker column, Roy must be the
broker. Place a / in that box.

Teacher Banker Chef Broker


Mark X3 / X3 X4

39
Zen / X1 X3 X1
Linda X2 X2 / X3
Roy X3 X4 X3 /

Mark is the banker, Zen is the teacher, Linda is the chef, and Roy is the
broker.

LESSON 2:
KENKEN PUZZLE

KenKen is an arithmetic-based logic puzzle that was invented by the


Japanese mathematics teacher Tetsuya Miyamoto in 2004. The noun “kenken” has
“knowledge” and “awareness” as synonyms. Hence, Kenken translates as knowledge
squared, or awareness squared.
In recent years, the popularity of KenKen has increased at a dramatic rate.
More than a million KenKen puzzle books have been sold, and KenKen puzzles now
appear in many popular nerwspapers, including the New York Times and the Boston
Globe.
KenKen puzzles are similar to Sudoku puzzles, but they also require you to
perform arithmetic to solve the puzzle.

Remember!

Rules for Solving KenKen Puzzle

For a 3 by 3 puzzle, fill in each box (square) of the grid with one of the
numbers 1, 2, or 3.
For a 4 by 4 puzzle, fill in each square of the grid with one of the
numbers 1, 2, 3, or 4.
For a n by n puzzle, fill in each square of the grid with one of the
numbers 1, 2, 3, …, n.

Grids range in size from a 3 by 3 up to a 9 by 9.


 Do not repeat a number in any row or column.
 The numbers in each heavily outlined set of squares, called cages,
must combine (in some order) to produce the target number in the top
left corner of the cage using the mathematical operation indicated.
 Cages with just one square should be filled in with the target number.
 A number can be repeated within a cage as long as it is not in the
same row or column.

40
Here is a 4 by 4 puzzle and its solution. Properly constructed puzzles have a
unique solution.

Basic Puzzle Solution Strategies

Single-Square Cages. Fill cages that consist of a single square with the target
number for that square.

Cages with Two Squares. Next examine the cages with exactly two squares. Many
cages that cover two squares will only have two digits that can be used to fill the
cage. For instance, in a 5 by 5 puzzle, a 20X cage with exactly two squares can only
be filled with 4 and 5 or 5 and 4.

Large or Small Target Numbers. Search for cages that have an unusually large or
small target number. These cages generally have only a few combinations of
numbers that can be used to fill the cage.

Examples:
In a 6 by 6 puzzle, a 120x cage with exactly three squares can only be filled
with 4, 5, and 6.

A 3+ cage with exactly two squares can only be filled with 1 and 2.

Duplicate Digit in a Cage. Consider a 3x cage. The digits 1, 1, and 3 produce a


product of 3; however, we cannot place the two 1s in the same row or the same
column. Thus the only way to fill the squares is to place the 3 in the corner of the L-
shaped caged as shown below. Remember: A digit can occur more than once in a
cage, provided that it does not appear in the same row or in the same column.

41
Remember the Following Rules

In an n by n puzzle, each row and column must contain every digit from 1 to n.

In a two-square cage that involves subtraction or division, the order of the


numbers in the cage is not important. For instance, a 3- cage with two squares could
be filled with 4 and 1 or with 1 and 4. A 3÷ cage with two squares could be filled with
3 and 1 or with 1 and 3.

LESSON 3
POLYA’S PROBLEM SOLVING STRATEGY

Ancient mathematicians such as Euclid and Pappus were interested in


solving mathematical problems, but they were also interested in heuristics, the study
of the methods and rules of discovery and invention. In the seventeenth century, the
mathematician and philosopher Rene Descartes (1596-1650) contributed to the field
of heuristics. He tried to develop a universal problem-solving method. Although he
did not achieve this goal, he did publish some of his ideas in Rules for the Direction
of the Mind and his better-known work Discourse de la Methode.
One of the foremost recent mathematicians to make a study of problem
solving was George Polya (1887-1985). He was born in Hungary and moved to the
United States in 1940. The basic problem solving strategy that Polya advocated
consisted of the following four steps.

Remember!
Polya’s Four-step Problem Solving Strategy
1. Understand the Problem
2. Devise a Plan
3. Carry out the plan.
4. Review the solution.

Understand the Problem. This part of Polya’s four-step strategy is often overlooked.
You must have a clear understanding of the problem. To help you focus on
understanding the problem, consider the following questions.
 Can you restate the problem in your own words?

42
 Can you determine what is known about these types of problems?
 Is there missing information that, if known, would allow you to solve the
problem?
 Is there extraneous information that is not needed to solve problem?
 What is the goal?

Devise a Plan. Successful problem solvers use a variety of techniques when they
attempt to solve a problem. Here are some frequently used procedures.
 Make a list of the known information.
 Make a list of information that is needed.
 Draw a diagram.
 Make an organized list that shows all the possibilities.
 Make a table or a chart.
 Work backwards.
 Try to solve a similar but simpler problem.
 Look for a pattern.
 Write an equation. If necessary, define what each variable represents.
 Perform an experiment
 Guess at a solution and then check your result.

Carry Out the Plan. Once you have devised a plan, you must carry it out.
 Work carefully.
 Keep an accurate and neat record of all your attempts.
 Realize that some of your initial plans will not work and that you may have to
devise another plan or modify your existing plan.

Review the Solution. Once you have found a solution, check the solution.
 Ensure that the solution is consistent with the facts of the problem.
 Interpret the solution in the context of the problem.
 Ask yourself whether there are generalizations of the solution that could apply
to other problems.

Example 7. Apply Polya’s Strategy (Make an organized list)

A baseball team won two out of their fast four games. In how many
different orders could they have two wins and two losses in four games?

Solution.

Understand the Problem

There are many different orders. They team may have won two straight
games and lost the last two (WWLL). Or maybe they lost the first two games
and won the last two (LLWW). Of course there are other possibilities, such as
WLWL.

43
Devise a Plan

We will make an organized list of all the possible orders. An organized list is a
list that is produced using a system that ensures that each of the different
orders will be listed once and only once.

Carry Out the Plan

Each entry in our list must contain two Ws and two Ls. We will use a strategy
that makes sure each order is considered, with no duplications. Once such
strategy is to always write a W unless doing so will produce to many. Ws or a
duplicate of one of the previous orders. If it is not possible to write a W, then
and only then do we write an L. This strategy produces the six different orders
shown below.
1. WWLL (Start with two wins)
2. WLWL (Start with one win)
3. WLLW
4. LWWL (Start with one loss)
5. LWLW
6. LLWW (Start with two losses)

Review the Solution

We have made an organized list. The list has no duplicates and the list
considers all possibilities, so we are confident that there are six different
orders in which a baseball team can win exactly two out of four games

Chapter
In items 1 to 7, useExercises
inductive reasoning to predict the next number in each list.

1. 4, 8, 12, 16, 20, 24, ________


2. 5, 11, 17, 23, 29, 35, _________
3. 3, 5, 9, 15, 23, 33, __________
4. 1, 8, 27, 64, 125, __________
5. 1, 4, 9, 16, 25, 36, 49, __________
6. 2, 7, -3, 2, -8, -3, -13, -8, -18, _________
7. 1, 5, 12, 22, 35, _________

In items 8 to 12, use inductive reasoning to decide whether each statement is


Correct or Wrong. Note: The numbers 1, 2, 3, 4, 5, … are called counting numbers
or natural numbers. Any counting number n divided by 2 produces a remainder of 0
or 1. If n ÷ 2 has a remainder of 0, then n is an even counting number. If n ÷ 2 has
a remainder of 1, then n is an odd counting number.

44
8. The sum of any two even counting numbers is always an even counting
number. __________________
9. The product of an odd counting numbers is always an odd counting number.
___________________
10. The product of two odd counting numbers is always an odd counting numbers
is always an odd counting number.
11. The sum of two odd counting numbers is always an odd counting number.
___________________
12. Pick any counting number. Multiply the number by 6. Add 8 to the product.
Divide the sum by 2. Subtract 4 from the quotient. The resulting number is
twice the original number.

In items 13 to 17 find a number that provides a counterexample to show that the


given statement is false.

1
13. For all numbers x, x > Answer: ________________________
x
x
14. For all numbers x, x + > x Answer: ________________________
x
15. For all numbers x, x3 ≥ x Answer: ________________________
16. For all numbers x, /x + 3/ = /x/ + 3 Answer: ________________________
17. For all numbers x, -x < x Answer: ________________________

18. Use deductive reasoning to show that the following procedure always
produces a number that is equal to the original number.
Procedure: Pick a number. Multiple the number by 6 and add 8. Divide the
sum by 2, subtract twice the original number, and subtract 4.

19. Use deductive reasoning to show that the following procedure always
produce the number 5.
Procedure: Pick a number. Add 4 to the number and multiply the sum by 3.
Subtract 7 and then decrease this difference by the triple of the original
number.

In items 20 to 23, determine whether the argument is an example of inductive


reasoning or deductive reasoning.

20. Samantha got an A on each of her first four math tests, so she will get an A
on the next math test. Answer: __________________________________
21. All books written by J.K. Rowling make the best-seller list. The book Harry
Potter and the Deathly Hallows is a J.K. Rowling book. Therefore, Harry
Potter and the Deathly Hallows made the bestseller list.
22. We had rain each day for the last five days, so it will rain today.
23. All amoeba multiply by dividing, I have named the amoeba shown in my
microscope Amelia. Therefore, Amelia multiplies by dividing.

45
Problem Solving:

Direction: Answer the following. Show all pertinent solutions.

1. Janet, Letty, Becky, and Rolly were recently elected as the new class officers
(president, vice president, secretary, treasurer) of the sophomore class at
Sunbeam College.

From the following clues, determine which position each holds:


a. Rolly is younger than the president but older than the treasurer.
b. Janet and the secretary are both the same age, and they are the
youngest members of the group.
c. Becky and the secretary are next-door neighbors.

President Vice-Pres Secretary Treasurer


Janet
Letty
Becky
Rolly

Answer: President: ______________ Secretary: ________________


Vice-Pres.:________________ Treasurer: ________________

2. Solve each of the following puzzles.

46
3. Using Polya’s Strategy, solve the following problem. Show your step-by-step
solution

Problem: A true-false quiz contains five questions. In how many ways can a
student answer the questions if the student answers two of the
questions with “false” and the other three with “true”?

Chapter 4:
Data Management
Introduction
T
he role of data management tools is important to further analyze and
interpret data. Utilizing these tools will greatly enhance the theories that might be
otherwise misunderstood.
This module deals with measures of central tendency, measures of
dispersion, measures of relative position, and normal distribution.

Learning Outcomes:
At the end of this chapter, you are expected to:

 Utilize various data management tools to process and manage


quantitative data;
 Calculate the measure of central tendency, measures of dispersion, and
measures of relative position, and standard score of a given set of data.
 Interpret data based on the result of computation.
 Appreciate the importance and application of measures of central
tendency, measures of dispersion, measures of relative position, and
normal distribution in real life situations.

47
LESSON 1:
MEASURES OF CENTRAL TENDENCY

A measure of central tendency is a summary statistic that represents the


center point or typical value of a dataset. It also referred to as the central location of a
distribution. There are three measures of central tendency - mean, median, and
mode. Choosing the best measure of central tendency depends on the type of data.

Mean for Ungrouped Data

The mean (also known as the arithmetic mean) is the most commonly used
measure of central position. It is the sum of measures divided by the number of
measures in a variable. It is symbolized as x (read as x bar). Mean is appropriate to
use when the distribution is at least interval scale.
To find the mean of ungrouped data, use the formula

Remember!
Ʃx
x=
n
where:
∑x = sum of entries n = number of entries

Example 1: The grades in Chemistry of 10 students are 87, 84, 85, 85, 86, 90, 79,
82, 78, 76. What is the average grade of the 10 students?

Solution:

87+84 +85+ 85+86+90 +79+82+78+76


x= = 83.2
10

Weighted Mean

Occasionally, we want to find the mean of a set of values wherein each value
or measurement has a different weight or degree of importance. We call this the
weighted mean and the formula for computing it is as follows:

Remember!
ƩxW
x=
∑W
where:
x = measurement or value W = weight

48
Example 2: Below are Maria’s subjects and the corresponding number of units and
grades she got for the first grading period. Compute her grade point
average.

Subject No. of Units (w) Grade (x)


Math 1.5 80
English 1.5 82
Filipino 1 83
Science 2 81
Social Studies 1 80
PEHM 1.5 85
THE 2 82

Solution:

x
 xW
W
80(1.5)  82(1.5)  83(1)  81(2)  80(1)  85(1.5)  82(2)

10.5

120  123  83  162  80  127.5  164



10.5

859.5

10.5

x  81.86

Therefore, Maria has the GPA of 81.86 for the first grading period.

Mean for Grouped Data

When the number of items in a set of data is too big, items are grouped for
convenience. The manner of computing for the mean of grouped data is given by the
formula:

Remember!
Ʃfx
x=
Ʃf
where:
x = class mark (midpoint of a class interval)
f= frequency of each class
49
Example 3: Compute the mean of the scores of the students in a Mathematics test.

Class Interval Frequency


46 - 50 1
41 - 45 5
36 - 40 11
31 - 35 12
26 - 30 11
21 - 25 5
16 - 20 2
11 -15 1

Solution: The frequency distribution for the data is given. The columns x and fx are
added.

Class Interval f x fx
46 - 50 1 48 48
41 - 45 5 43 215
36 - 40 11 38 418
31 - 35 12 33 396
26 - 30 11 28 308
21 - 25 5 23 115
16 - 20 2 18 36
11 - 15 1 13 13
Ʃf =48 Ʃfx=1,549

Ʃfx
x=
n

1,549
x=
48

x=32.27

The mean score is 32.27.

Example 4: Solve for the mean gross sale of Aling Mely’s Sari-sari Store for one
month.

Sales in Pesos Frequency


4,501 - 5,000 3
4,001 - 4,500 4
3,501 - 4,000 6
3,001 - 3,500 5

50
2,501 - 3,000 7
2,001 - 2,500 3
1,501 - 2,000 1
1,001 - 1,500 1

Solution: The frequency distribution for the data is given below. The columns x and
fx are added.

Sales in Pesos f x fx
4,501 - 5,000 3 4,750 14,250
4,001 - 4,500 4 4,250 17,000
3,501 - 4,000 6 3,750 22,500
3,001 - 3,500 5 3,250 16,250
2,501 - 3,000 7 2,750 19,250
2,001 - 2,500 3 2,250 6,750
1,501 - 2,000 1 1,750 1,750
1,001 - 1,500 1 1,250 1,250
Ʃf =30 Ʃfx=99,000

Ʃfx
x=
n

99,000
x=
30

x=3,300

The mean gross sale is Php 3,300.

Median for Ungrouped Data

The median is the middle entry or term in a set of data arranged in either
increasing or decreasing order. The median is a positional measure. Thus, the values
of the individual measures in a set of data do not affect it. It is affected by the number
of measures and not by the size of the extreme values. This measure is appropriate
to use when the distribution is at least ordinal scale since ranking of the data is
involved.

To find the median of a given set of data, take note of the following:

1. Arrange the data in either increasing or decreasing order.


2. Locate the middle value. If the number of cases is odd, the middle values is
the median. If the number of cases is even, take the arithmetic mean of the
two middle measures.

51
Example 5: The number of books borrowed in the library from Monday to Friday last
week were 58, 60, 54, 35, and 97 respectively. Find the median.

Solution: Arrange the number of books borrowed in increasing order.

35, 54, 58, 60, 97

The median is 58.

Example 6: Cora’s quizzes for the second quarter are 8, 7,6, 10, 9, 5, 9, 6, 10, and
7. Find the median.

Solution: Arrange the scores in increasing order.

5, 6, 6, 7, 7, 8, 9, 9, 10, 10

Since the number of measures is even, then the median is the average of
the two middle scores.

78
Md   7 .5
2

Median of Grouped Data

To find the median of grouped data, identify first the median class, the class
interval holding the median. Since the median divides the distribution into two equal
parts, first get 50% of the total number of cases or scores. Then identify the interval
containing the score where 50% of the cases would fall below this value.

In computing for the median of grouped data, the following formula is used:

Remember!

( )
Ʃf
−cf
2
Md=lbmc + i
f mc
where:
lb mc = true lower limit or lower-class boundary of the median class
cf = cumulative frequency of the lower class next to the median class
f mc = frequency of the median class
f = frequency of each class;
i = class size

Ʃf
The median class is the class that contains the th quantity. The computed
2
median must be within the median class.

Example 7: Compute the median of the scores of the students in a Mathematics test.

52
Class Interval Frequency
46 - 50 1
41 - 45 5
36 - 40 11
31 - 35 12
26 - 30 11
21 - 25 5
16 - 20 2
11 -15 1

Solution: The frequency distribution for the data is given below. The columns for lb
and “less than” cumulative frequency are added.

Class Interval f lb <cf


46 - 50 1 45.5 48
41 - 45 5 40.5 47
36 - 40 11 35.5 42
31 - 35 12 30.5 31
26 - 30 11 25.5 19
21 - 25 5 20.5 8
16 - 20 2 15.5 3
11 - 15 1 10.5 1
Ʃf =48
Ʃf 48
Since = =24 , the 24th quantity is in the class 31 - 35. Hence, the
2 2
median class is 31 - 35.

( )
Ʃf
−cf
2
Md=lbmc + i
f mc

( )
48
−19
2
Md=30.5+ 5
12

Md=30.5+32.08

Md=32.58

The median score is 32.58.

Example 8: Solve for the median gross sale of Aling Mely’s Sari-sari Store for one
month.

Sales in Pesos Frequency


4,501 - 5,000 3
4,001 - 4,500 4
3,501 - 4,000 6

53
3,001 - 3,500 5
2,501 - 3,000 7
2,001 - 2,500 3
1,501 - 2,000 1
1,001 - 1,500 1

Solution: The frequency distribution for the data is given below. The columns for lb
and “less than” cumulative frequency are added.

Sales in Pesos f lb <cf


4,501 - 5,000 3 4,500.5 30
4,001 - 4,500 4 4,000.5 27
3,501 - 4,000 6 3,500.5 23
3,001 - 3,500 5 3,000.5 17
2,501 - 3,000 7 2,500.5 12
2,001 - 2,500 3 2,000.5 5
1,501 - 2,000 1 1,500.5 2
1,001 - 1,500 1 1,000.5 1
Ʃf =30

Ʃf 30
Since = =15, the 15th quantity is in the class 3,001- 3,500. Hence, the
2 2
median class is 3,001- 3,500.

( )
Ʃf
−cf
2
Md=lbmc + i
f mc

( )
30
−12
2
Md=3000.5+ 500
5

Md=3000.5+300

Md=3,300.5

The median gross sale is Php 3,300.5

Mode for Ungrouped Data

The mode is another measure of position. The mode is the measure or value
which occurs most frequently in a set of data. It is the value with the greatest
frequency. Mode is appropriate to use when the variable measured is in the nominal
scale.

To find the mode for a set of data:

54
1. select measure that appears most often in the set;
2. if two or more measures appear the same number of items, and the
frequency they appear is greater than any of other measures, then each of
these values is a mode;
3. if every measure appears the same number of items, then the set of data
has no mode.

Example 9: The shoe size of 10 randomly selected students in a class are 6, 5, 4, 6,


4.5, 5, 6, 7, 7 and 6. What is the mode?

Solution: The mode is 6 since it is the shoe size that occurred the most number of
times.

Example 10: The sizes of 9 classes in a certain school are 50, 52, 55, 50, 51, 54, 55,
53 and 54.

Solution: The modes are 54 and 55 since the two measures occurred the same
number of times. The distribution is bimodal.

Mode for Grouped Data

The mode pf grouped data can be approximated using the following formula:

Remember!
Mo=Lbmo +
( D1
D 1+ D 2)i

where:

LLmo = true lower limit or lower-class boundary of the modal class


D1 = is the difference between the frequencies of the modal class
and the next lower class
D2 = is the difference between the frequencies of the modal class
and the next upper class
i = the class size

The modal class is the class with the highest frequency. If binomial classes
exist, any of these classes may be considered as modal class.

Examples 11: Compute the mode of the scores of the students in a Mathematics
test.

55
Class Interval Frequency
46 - 50 1
41 - 45 5
36 - 40 11
31 - 35 12
26 - 30 11
21 - 25 5
16 - 20 2
11 -15 1

Solution: The frequency distribution for the data given below. The column for lb is
added.

Class Interval f lb
46 - 50 1 45.5
41 - 45 5 40.5
36 - 40 11 35.5
31 - 35 12 30.5
26 - 30 11 25.5
21 - 25 5 20.5
16 - 20 2 15.5
11 - 15 1 10.5

Since class 31 - 35 has the highest frequency, the modal class is 31 -


35.

Mo=Lbmo +
( D1
D 1+ D 2)i

Mo=30.5+ ( 1+11 )5
Mo=30.5+2.5

Mo=33

The mode score is 33.

Example 12. Solve for the modal gross sale of Aling Mely’s Sari-sari Store for one
month.

Sales in Pesos Frequency


4,501 - 5,000 3
4,001 - 4,500 4
3,501 - 4,000 6
3,001 - 3,500 5

56
2,501 - 3,000 7
2,001 - 2,500 3
1,501 - 2,000 1
1,001 - 1,500 1

Solution: The frequency distribution for the data is given below. The columns for lb
is added.

Sales in Pesos f lb
4,501 - 5,000 3 4,500.5
4,001 - 4,500 4 4,000.5
3,501 - 4,000 6 3,500.5
3,001 - 3,500 5 3,000.5
2,501 - 3,000 7 2,500.5
2,001 - 2,500 3 2,000.5
1,501 - 2,000 1 1,500.5
1,001 - 1,500 1 1,000.5

Since the class 2,501 - 3,000 has the highest frequency, the modal class is
2,501 - 3,000.

Mo=Lbmo +
( D1
D 1+ D 2)i

Mo=2,500.5+ ( 4+4 2 )500


Mo=2,500.5+333.33

Mo=2,833.33

The modal gross sale is Php 2,833.33.

LESSON 2:
MEASURES OF DISPERSION

The measures that describe the degree of spread of the data are called
“measure of dispersion” or “measure of variability” or “measure of spread”. This
measure is used to determine how scattered the values are in the distribution. In this
topic, we will consider four measures of dispersion, namely: range, average
deviation, variance, and standard deviation.

Range for Ungrouped Data

The range is the simplest measure of variability. It is the difference between


the largest ad smallest measurement. To determine the range of ungrouped data, the
formula is

57
Remember!
R=H−L
where:
H = Highest measure L= Lowest measure

The main advantage of the range is that it does not consider every measure
in the data.

Example 13: Consider the four data sets presented below. Find the range of each
data set.

Data Set
Data Set 1 11 12 13 14 15
Data Set 2 13 14 15 17 19
Data Set 3 10 15 18 20 22
Data Set 4 21 23 25 27 30
Solution:

Data Set 1: R=H – L Data Set 2: R=H – L


` R=15 – 11 R=19 – 13
R=4 R=6

Data Set 3: R=H – L Data Set 4: R=H – L


` R=22 – 10 R=30 – 21
R=12 R=9

Comparing the data sets, Data Set 1 has the least variation because it has
the smallest value of R. On the other hand, Data Set 3 has the most variation
because it has the largest value of R.

Range for Grouped Data

The range of a grouped data is simply the difference between the upper class
boundary of the top interval an lower class boundary of the bottom interval.

Example 14: Find the range of the scores in Midterm Exam of BEEd First Year
Students.

Class Interval Frequency


46 - 50 1
41 - 45 10
36 - 40 10
31 - 35 16
26 - 30 9
21 - 25 4

Solution:
Upper class boundary (UCB) = 50.5

58
Lower class boundary (LCB) = 20.5

UCB - LCB = 50.5 - 20.5 = 30

Average Deviation for Ungrouped Data

The Average Deviation (AD) is a measure of absolute dispersion that is


affected by every individual score. It is the mean of the absolute deviations of the
individual scores from the mean of all the scores.
A large average deviation would mean that a set of scores is widely dispersed
about the mean, while a small average deviation would imply that the set of scores is
closer to the mean.
The formula of average deviation for ungrouped data is:

Remember!
AD=
∑ ¿ x−x∨¿ ¿
n−1

where: Ʃ = symbol for summation n = total number of scores


x = individual score x = mean of all scores

To be able to apply the formula, the following steps can be observed:

1. Compute the mean from the given scores.


2. Subtract the mean from the individual scores to get the deviation. That is,
x−x
3. Get the absolute value of each deviation.
4. Get the sum of the absolute deviation and divide it by (n-1), where n is the
total number of scores. The quotient is the average deviation.

Example 15. The raw scores of eight students in Statistics are given as follows: 17,
17, 26, 28, 30, 30, 31, and 37. Compute the average deviation.

Solution:

Score Deviation Absolute Deviation


x x−x ¿ x−x∨¿
17 -10 10
17 -10 10
26 -1 1
28 1 1
30 3 3
30 3 3
31 4 4
37 10 10
x=27 Ʃ∨x−x ∨¿ 42

59
AD=
∑ ¿ x−x∨¿ ¿
n−1

42
AD=
8−1

42
AD=
7

AD=6

Example 16. The scores of nine students in Psychology are given as follows: 15, 19,
20, 24, 28, 30, 32, 32, and 40. Calculate the average deviation.

Solution:

Score Deviation Absolute Deviation


x x−x ¿ x−x∨¿
15 -11.67 11.67
19 -7.67 7.67
20 -6.67 6.67
24 -2.67 2.67
28 1.33 1.33
30 3.33 3.33
32 5.33 5.33
32 5.33 5.33
40 13.33 13.33
x=26.27 Ʃ∨x−x ∨¿ 57.33

AD=
∑ ¿ x−x∨¿ ¿
n−1

57.33
AD=
9−1

57.33
AD=
8

AD=7.17

The computed average deviation (A.D.) of scores in Statistics is 6 while test


scores in Psychology is 7.17. This can be interpreted as the scores in Statistics are
less dispersed or closely distributed near the mean (homogeneous) while the scores
in Psychology are more dispersed away from the mean (heterogeneous).

Average Deviation for Grouped Data

For the grouped data or scores organized in the form of frequency


distribution, the average deviation is computed as follows:

Remember! 60
AD=
∑ f i ∨xi −x∨¿ ¿
n−1

where: Ʃ = symbol for summation n = total number of scores


xi = midpoint of the ith class interval x = mean of all scores
fi = frequency of the ith class interval

The steps in determining the average deviation of grouped data are as


follows:

1. Find the mean of the frequency distribution.


2. Get the midpoint of each class interval.
3. Subtract the mean from the midpoint of each class interval to get the
deviations and then, take their absolute values.
4. Multiply the frequency of each class interval to the corresponding absolute
deviation and add all the products to get ∑ f i∨x i−x∨¿ ¿.
5. Divide the sum by (n - 1), where n is the total number of frequencies. The
quotient is the average deviation.

Example 17. Below is a frequency distribution of test scores of 50 students in


Assessment of Learning. Find the average deviation.

Class Interval Frequency


36 – 40 7
31 – 35 10
26 – 30 5
21 – 25 14
16 – 20 6
11 – 15 8
Ʃf =50

Solution: The mean of this distribution is x=25.40 .

Class fi xi x i−x ¿ x i−x∨¿ ¿ f i∨x i−x∨¿ ¿


Interval
36 – 40 7 38 12.60 12.60 88.2
31 – 35 10 33 7.60 7.60 76
26 – 30 5 28 2.60 2.60 13
21 – 25 14 23 -2.40 2.40 33.6
16 – 20 6 18 -7.40 7.40 44.4
11 – 15 8 13 -12.40 12.40 99.2
x=25.40 n=50 Ʃ f i∨x i−x ∨¿=354.4 ¿

AD=
∑ f i ∨xi −x∨¿ ¿
n−1

61
354.4
AD=
50−1

354.4
AD=
49

AD=7.23

We say that the scores deviate from the mean of 25.40 by an average of 7.23
units.

Variance for Ungrouped Data

Another way to avoid a sum of zero for the deviation scores is to square each
deviation score and get the average of all squared deviation scores. The resulting
measure is called “variance” which has a squared unit. In symbol, s2.

To compute the variance of ungrouped data, the following formula may be


used

Remember! 2
Ʃ(x−x )
s2=
n

where: x = individual score n = total number of scores


x = mean of all scores

To be able to apply the formula, the following steps ca be observed:

1. Arrange the values in column from lowest to highest.


2. Compute the mean of the distribution.
3. Determine the deviation (d= x−x ¿.
4. Square the deviations.
5. Get the sum of the squared deviations.
6. Divide the sum by the total number of cases. The quotient is the variance.

Example 18. Consider the data set below. Compute the variance of each data set.

Data Set
Data Set 1 13 16 14 10 15
Data Set 2 22 25 23 27 29
Solution:

Data Set 1:

Score Deviation Squared


x x−x Deviation
2
(x−x )
10 -3.6 12.96

62
13 -0.6 0.36
14 0.4 0.16
15 1.4 1.96
16 2.4 5.76
x=13.6 2
Ʃ(x−x ) =21.2

2
2 Ʃ(x−x )
s=
n

2 21.2
s=
5
2
s =4.24

Data Set 2:

Score Deviation Absolute Deviation


x x−x (x−x )
2

22
Ʃf ( x−x)-3.2 10.24
2
2
23s = -2.2 4.84
25 n -0.2 0.04
27 1.8 3.24
29 frequency 3.8
Where: f = class 14.44
x=25.2
x = class mark
2
Ʃ(x−x ) =32.8
x = mean of the distribution
2
Ʃ(x−x ) n = total number of cases or scores
s2=
n

2 32.8
s=
5
2
s =6.56

Variance for Grouped Data

When the data are presented in frequency distribution, the following formula
must be used

Remember! 2
2 Ʃf ( x−x)
s=
n−1

where: x = class mark n = total number of scores


x = mean of all scores f = frequency

63
To be able to apply the formula, the following steps ca be observed:

1. Compute the mean of the distribution.


2. Determine the deviation (d= x−x ¿.
3. Square the deviations.
4. Multiply the deviation by its corresponding frequency.
5. Add the results in step 4.
6. Divide the sum by n – 1. The quotient is the variance.

Example 19: Below is a frequency distribution of test scores of 50 students in


Assessment of Learning. Find the variance of the scores.

Class Interval Frequency


36 – 40 7
31 – 35 10
26 – 30 5
21 – 25 14
16 – 20 6
11 – 15 8
Ʃf =50

Solution: The mean of this distribution is x=25.40 .

Class 2 2
f x x−x (x−x ) f (x−x)
Interval
36 – 40 7 38 12.60 158.76 1,111.32
31 – 35 10 33 7.60 57.76 577.6
26 – 30 5 28 2.60 6.76 33.8
21 – 25 14 23 -2.40 5.76 80.64
16 – 20 6 18 -7.40 54.76 328.56
11 – 15 8 13 -12.40 153.76 1,230.08
x=25.40 Ʃf =50 2
Ʃ f (x−x ) =3,362

2
Ʃf ( x−x)
s2=
n−1

2 3,362
s=
49

2
s =68.61
Example 20: Consider the frequency distribution below. Calculate the variance of the
distribution.

Class Interval Frequency (f)


33 – 37 6
28 – 32 9
23 – 27 12
18 – 22 8

64
13 – 17 10

Solution: The mean of this distribution is x=24.22.

Class 2 2
f x x−x (x−x ) f (x−x)
Interval
33 – 37 6 35 10.78 116.2084 697.2504
28 – 32 9 30 5.78 33.4084 300.6756
23 – 27 12 25 0.78 0.6084 7.3008
18 – 22 8 20 -4.22 17.8084 142.4672
13 – 17 10 15 -9.22 85.0084 850.084
x=24.22 Ʃf =45 2
Ʃ f ( x−x ) =1,997.778

2
2 Ʃf ( x−x)
s=
n−1

2 1,997.778
s=
44

2
s =45.40

Standard Deviation for Ungrouped Data

Recall that, in the computation of the variance, the deviation was squared.
This implies that the variance is expressed in squared units. Extracting the square
root of the value of the variance will give the value of the standard deviation. In
symbol, s.

To take the standard deviation of ungrouped data, extract the square root of
the variance. In mathematical formula,

Remember!
s=

Ʃ(x−x )2
n

Example 21. Consider the data set below. Compute the standard deviation of each
data set.

Data Set
Data Set 1 13 16 14 10 15
Data Set 2 22 25 23 27 29
Solution:

Data Set 1:

65
By example 18, the variance of Data Set 1 is s2=4.24 . Hence, the
standard deviation is

s=

Ʃ(x−x )2
n

s= √ 4.24

s=2.06

Data Set 2:

By example 18, the variance of Data Set 2 is s2=6.56 Hence, the


standard deviation is

s=
√ Ʃ(x−x )2
n

s= √ 6.56

s=2.56

On the basis of the obtained standard deviation, we say that the scores in
Data Set 1 deviate from the mean by 2.06 units, on the everage. For Data Set 2, the
scores deviate from the mean by an average of 2.56 units.

Standard Deviation for Grouped Data.

To take the standard deviation of grouped data, extract the square root of the
variance. In mathematical formula,

Remember!
s=
√Ʃ(x−x )2
n−1

Example 22: Below is a frequency distribution of test scores of 50 students in


Assessment of Learning. Find the standard deviation of the scores.

Class Interval Frequency

66
36 – 40 7
31 – 35 10
26 – 30 5
21 – 25 14
16 – 20 6
11 – 15 8
Ʃf =50
Solution:

By example 19, the variance is s2=68.61 . Hence, the standard deviation is

s=
√ Ʃf (x −x)2
n

s= √ 68.61

s=8.28

The scorese deviate from the mean by an average of 8.28 units.

Example 23: Consider the frequency distribution below. Calculate the variance of the
distribution.

Class Interval Frequency (f)


33 – 37 6
28 – 32 9
23 – 27 12
18 – 22 8
13 – 17 10

Solution:

By example 20, the variance of is s2=45.40. Hence, the standard deviation is

s=

Ʃf (x −x)2
n−1

s= √ 45.40

s=6.74

The scorese deviate from the mean by an average of 6.74 units.

LESSON 3:
67
MEASURES OF RELATIVE POSITION
Measures of Relative Position also referred as quantiles are descriptive
measures that locate the relative position of a score in relation to the rest in a given
distribution. There are three measures of relative position, namely: the percentile,
deciles, and quartiles.

Quartiles (Q)

Quartiles are the score points which divides the distribution into four equal
parts. Each set of observations has 3 quartiles and are denoted by Q1, Q2, and Q3.

Q1 Q2 Q3

a. 25% of the distribution has a value ≤ Q 1 (lower quartile or the first


quartile).
b. 50% of the distribution has a value ≤ Q2 (median or middle quartile).
c. 75% of the distribution has a value ≤ Q3 (upper quartile or the last
quartile).
Deciles (D)

Quartiles are the score points which divides the distribution into ten equal
parts. Each set of observations has 9 deciles and are denoted by D1, D2, D3, …D9.

D1 D2 D3 D4 D5 D6 D7 D8 D9

a. 10% of the distribution has a value ≤ D1 (first decile).


b. 50% of the distribution has a value ≤ D5 (median or fifth decile).
c. 90% of the distribution has a value ≤ D9 (9th decile).
Percentiles (P)

Percentiles are the score points which divides the distribution into one -
hundred equal parts. Each set of observations has 99 percentiles and are denoted by
P1, P2, P3, …P99.

P10 P20 P30 P40 P50 P60 P70 P80 P90


0
a. 10% of the distribution has a value ≤ P10 (10th percentile).
b. 50% of the distribution has a value ≤ P50 (median or %0th percentile).

68
c. 90% of the distribution has a value ≤ P90 (90th percentile).
Relationship Among Percentile, Decile, and Quartile

 P10 = D1
 P20 = D2
 P25 = Q1
 P50 = D5 = Q1 = median
 P75 = Q3
 P90 = D9

Quantiles for Ungrouped Data

To compute the quantiles of ungrouped data, we will be using the (n + 1)


method, where n represents the total number of scores in a data set.

If (n+1) x p= jg where j is the integral part and g is the fractional part of the
product, then

Remember!

PV =X j + g( X j+ 1−X j)

Thus, PV is the number in the jth position (Xj) of the ordered data plus g
multiplied by the difference between the succeeding value (Xj+1) and (Xj).

Example 24: Find Q1, D5, P80, and P99 for the following data:

45 67 78 55 88 90 56 68 99 40

65 70 86 99 59 75 45 84 69 50

Solution: Arrange first the data from lowest to highest

40 45 45 50 55 56 59 65 67 68

69 70 75 78 84 86 88 90 99 99

Solving for Q1.

Since Q1 = P25, we get 25% (20+1) = 5.25, which mean that j = 5


and g = .25 which means that Q1 is the 5th score (55) plus 0.25 of
the difference between the 6th score (56) and the 5th score (55).
Hence,

PV =X j + g( X j+ 1−X j)
Q1=55+.25 (56−55)
Q1=55+.25

69
Q1=55.25
Hence, 25% of the scores in the distribution are below 55.25.

Solving for D5.

Since D5 = P50, we get 50% (20+1) = 10.5, which mean that j = 10


and g = .50 which means that D 5 is the 10th score (68) plus 0.50 of
the difference between the 11th score (69) and the 5th score (68).
Hence,

PV =X j + g( X j+ 1−X j)
D5=68+ .50(69−68)
D5=68+ .50
D5=68.50

Hence, 50% of the scores in the distribution are below 68.50.

Solving for P80.

For P80, we get 80% (20+1) = 16.8, which mean that j = 16 and g
= .80 which means that P 80 is the 16th score (86) plus 0.80 of the
difference between the 17th score (88) and the 16th score (86).
Hence,

PV =X j + g( X j+ 1−X j)
P80=86 +.80(88−86)
P80=86 +1.60
P80=87.60

Hence, 80% of the scores in the distribution are below 87.60.

Solving for P99.

For P99, we get 99% (20+1) = 20.79. Thus, P 99 is the score which
is .79 of the way from the 20 th score to the next score. Since we do
not have a score beyond the 20th score we take the 20th score as
the value of P99. Therefore, P99 = 99.

Quantiles for Grouped Data

To compute the quantiles of grouped data, similar computation of the median


will be followed. Since Px is the value below which x% of the total number of cases
n
lies, we can revise the formula for median by changing with x%(n). Therefore,
2

70
Remember!

( )
n
−F b
2
P x =¿+ c
f
where:
LL = true lower limit of the class interval containing Px
Fb = the sum of all frequencies below the intervals
containing Px (or the <cf directly below the intervals
containing Px)
f = frequency of the intervals containing Px
c = class size;
n = total number of cases

Example 25. Consider the frequency distribution below. Find Q1, D4, and P90.

Class Interval F <cf


35.5 – 40.5 7 50
30.5 – 35.5 10 43
25.5 – 30.5 5 33
20.5 – 25.5 14 28
15.5 – 20.5 6 14
10.5 – 15.5 8 8
Ʃf =50

Solution:

Solving for Q1.

Since Q1 = P25, we first get 25%(n) to determine the interval class


containing Q1. Note that 25% (50) = 12.5. With reference to the “<cf”
column, 12.5 is between 8 and 14, so, the interval 15.5 – 20.5 contains
P25. Thus, with reference to this interval, we have LL = 15.5; Fb = 8; f =
6; and c = 5.

Px=¿+ ( f )
x % (n)−F b
c

Q 1=15.5+ ( 12.5−8
6 )5
Q1=15.5+3.75

71
Q1=19.25

Hence, 25% of the scores in the distribution are below 19.25.

Solving for D4.

Since D4 = P40, we first get 40%(n) to determine the interval class


containing D4. Note that 40% (50) = 20. With reference to the “<cf”
column, 20 is between 14 and 28, so, the interval 20.5 – 25.5 contains
D4. Thus, with reference to this interval, we have LL = 20.5; Fb = 14; f =
14; and c = 5.

Px=¿+ ( x % (n)−F
f )c
b

D4 =20.5+ ( 20−14
14 )
5

D4 =20.5+2.14

D4 =22.64

Hence, 40% of the scores in the distribution are below 22.64.

Solving for P90.

For P90, we first get 90%(n) to determine the interval class containing
P90. Note that 90% (50) = 45. With reference to the “<cf” column, 45 is
between 43 and 50, so, the interval 35.5 –40.5 contains P 90. Thus, with
reference to this interval, we have LL = 35.5; Fb = 43; f = 7; and c = 5.

Px=¿+ ( x % (n)−F b
f
c)
P90=35.5+ ( 45−43
7 )
5

P90=35.5+1.43

P90=36.93

Hence, 90% of the scores in the distribution are below 36.93.

Box Plots or “Box Whiskers Diagram”

72
A box plot, also known as Box Whiskers Diagram is a visual presentation of
a summary of a data set. This is obtained by plotting the values of the five descriptive
statistics of the data which are the smallest value, the lower quartile (Q 1), the median
(Q2), the upper quartile (Q3), and the largest value.

To construct a box plot, the following procedures must be considered:

1. Draw a box that spans from Q1 to Q3.


2. Draw line segment in the box that marks the median Q2.
3. Draw line segment (called whiskers) that extend from the box to the
smallest and largest values of the data.

Example 26: Construct a box plot for the following data:

40 45 45 50 55 56 59 65 67 68

69 70 75 78 84 86 88 90 99 99

Solution:

Based on the data, we have the following five summary values:

1. Smallest value: 40
2. Q1 = 55.25
3. Q2 = 68.50
4. Q3 = 85.5
5. Largest value: 99
The resulting box plot is:
S Q1 Q2 Q3 L

30 40 50 60 70 80 90 100

LESSON 4:
NORMAL DISTRIBUTION

Normal Distribution

The most common distribution we see in statistics is the normal distribution.


This is also known as the Gaussian distribution or the “bell curve”. Normal distribution
is a family curves determined by parameters µ (mean) and σ (standard deviation).

Mean = Median = Mode


Symmetrical sides
73

Asymptotic tail
The properties of the normal distribution are as follows:

1. It is bell – shaped and is symmetric with respect to the vertical line that
passes through the highest point of curve.
2. It is unimodal and the mean, median and mode are equal.
3. It is asymptotic with respect to the baseline, which means that the tails of
the distribution get closer and closer to the baseline without crossing the
baseline.
4. The total area under the curve and above the baseline is always equal to
1.0.

Empirical Rule

Because the under the normal curve and above the baseline is 1.0, we
consider the normal curve as the graphic picture of the proportion of scores in a
distribution. We state below a common property of all normal curves with a given
mean µ and standard deviation σ. This property is called the empirical rule which
highlights one interpretation of the standard deviation as a concept of “distance”.

Source: statistics – made – easy.com

If a given set of data is assumed to be normally distributed with a given mean


µ and standard deviation σ, then

a. about 68.27% of all the cases are expected to fall between µ - σ and µ +
σ.
b. about 95.45% of all the cases are expected to fall between µ - 2σ and µ +
2σ.

74
c. about 99.73% of all the cases are expected to fall between µ - 3σ and µ +
3σ.
Example 27: Suppose the first-year college class consisting of 120 students posted
a mean score of 70 with a standard deviation of 9 in their final exam in
Math. Assuming that the scores are continuously and normally
distributed,

a. how many students are expected to score between 61 and 79?


b. within what two scores do we expect 95% of the students to
score?
Solution:

a. Note that 61 = 70 – 9 = µ - σ. While 79 = 70 + 9 = µ + σ. According to the


empirical rule, 68% of the scores are expected to fall between µ - σ and µ
+ σ. Hence, about 68% (120) ≈ 82 pupils are expected to score between
61 and 79.

b. Again, from the empirical rule, we expect about 95% of the scores to fall
between the values µ - 2σ and µ + 2σ. Since, µ - 2σ = 70 – 2(9) = 52 and
µ + 2σ = 70 + 2(9) = 88, then, about 95% of the pupils are expected to
score between 52 and 88.

Standard Scores (z-scores)

The standard score is the distance of the score from the mean in terms of the
standard deviation. It tells how many standard deviations the observed value lies
above or below the mean of its distribution. The standard score is useful in
comparing observed values from different distributions. To be able to find areas
under the normal curve, observed values must first be converted into standard
scores, and these would help solve statistical problems.

To change an observed value into standard score, you use the following
equation:

Remember!
x−x
z=
s

where: x = raw score / observed value s = standard score


x = mean

Note: A positive z-score will mean that the score/observed value is above
the mean.
A negative z-score will mean that the score/observed value is below
the mean.

75
Example 28. In a given distribution, the mean is 65 and the standard deviation is 6.
Find the corresponding standard score of:

a. 68 b. 59

Solution:

a. The corresponding standard score of 68 is

x −x 68−65 3
z 68= = = =0.5
s 6 6

which means that 68 is 0.5 standard deviation above the mean.

b. The corresponding z-score of 59 is

x −x 59−65 −6
z 59= = = =−1.0
s 6 6

which means that 59 is 1 standard deviation below the mean.

Example 29: On the final examination in Math, the mean grade was 82 and the
standard deviation was 8. In English, the mean grade was 86 and the
standard deviation was 10. Joseph scored 88 in Math and 92 in
English. In which subject was his standing higher?

Solution: The first that has to be done us change the scores into standard scores.

For English

x−x 92−86 6
zE = = = =0.6
s 10 10

For Math

x−x 88−86 6
zM = = = =0.75
s 8 8

His standing in Math was higher than his standing in English. He was 0.6
standard deviation above the mean in English and 0.75 standard deviation
above the mean in Math.

Standard Normal Distribution

A normal distribution can be converted into a standard normal distribution


by obtaining the z value. A z- value is the signed distance between a selected value,
designated x, and the mean, μ, divided by the standard deviation. It is also called as

76
z scores, the z statistics, the standard normal deviates, or the standard normal
values. In terms of formula:

Remember!
x−μ
z=
σ

where: x = raw score / observed value σ = standard deviation


μ = mean of the distribution

The normal distribution property allows to compute a probability problem


concerning x into one corniness z. To determine the probability that x lies in a given
interval, converting the interval to a z scale and then compute the probability by using
the standard normal distribution table.

77
Example 30. Find the area under the standard normal curve between the mean and
each given value of z:

a. z = -1.33
b. z = 1.75

Solution:

a. To find the area between the mean z = 0 and z = -1.33, we read the z
value of 1.3 on the first column, then the z value of 0.03 on the first row of
Table 1. The intersection of the identified row and column yields the
number 0.4082.

0.4082

-1.33 0

Thus, the area from the mean up to the value of z = -1.33 is 0.4082 or
40.82%

b. For z = 1.75, we read the z value of 1.7 on the first column, then the z
value of 0.05 on the first row. The intersection row and column yield the
number 0.4599 or 45.99%

0.4599

Thus, the area from the mean up to the value of z = 1.75 is 0.4599 or
45.99%

78
Example 31: Find the area under the standard normal curve

a. to the left of z = 2.0


b. to the right of z = -1.0
c. to the right of z = 1.96
d. to the left of z = - 2.65
e. between z = 1.5 and z = 2.75
f. between z = -1.0 and z = 2.0

Solution:

a. The area to the left of z = 2.0 includes the area from z = 0 and z = 2.0 plus
half of the entire area under the normal curve. From the table, the area
from the mean up to z = 2.0 is 0.4772. Therefore, the entire area to the
left of z = 2.0 is 0.5 + 0.4772 = 0.9772 or 97.72%

0.9772

0 2.0
b. The area to the right of z = -1.0 includes the area from the mean down to
z = -1.0 plus half of the entire area under the normal curve. By symmetry,
the area from the mean down to z = -1.0 is equal to the area from the
mean up to z = 1.0 which is 0.3414. Thus, the entire area to the right of z
= -1.0 is 0.5 + 0.3414 = 0.8414 or 84.14%

0.8414

-1.0 0

c. To find the area to the right of z = 1.96, we first note that the area from the
mean to the entire right is 0.5. If we subtract the area from the mean up to
z = 1.96 from 0.5, we get the desired area to the right of z = 1.96. Using
the normal table, the area from the mean up to z = 1.96 is 0.4750.
Therefore, the area to the right of z = 1.96 is 0.5 – 0.4750 = 0.025 or
2.5%.

0.025

0 1.96

79
d. To find the area to the left of z = -2.56 is equal to the area to the right of z
= 2.56 by symmetry. Using the normal table, the area from the mean up to
z = 2.56 is 0.4960. Therefore, the area to the left of z = -2.56 is 0.5 –
0.4960 = 0.004 or 0.4%.

0.004

e. To find the area between z = 1.5 -2.56


and z = 2.75, we get the area from the
0
mean up to z = 2.75, then subtract the area from the mean up to z = 1.5.
Using the normal table, the area from the mean up to z = 2.75 is 0.4970
while the area from the mean up to z = 1.5 is 0.4332. therefore, the
desired area is given by 0.4970 – 0.4332 = 0.0638 or 6.38%.

0.0638

0 1.56 2.75

f. The area from z = - 1.0 to z = 2.0 can be obtained by adding the area
from the mean down to z = - 1.0 and the area from the mean up to z =
2.0. By symmetry, the area from the mean down to z = - 1.0 is equal to
the area from the mean down to z = 1.0 which is 0.3414. Also, the area
from the mean up to z = 2.0 is 0.4772. Therefore, the desired area is
given by 0.3414 + 0.4772 = 0.8186 or 81.86%

0.8186

- 1.0 0 2.0

Application of Normal Distribution

Example 32. The average PAG-IBIG salary loan for RFS Pharmacy Inc. Employees
is ₱23,000. If the debt is normally distributed with a standard deviation
of ₱2,500, find the probability that the employee owes less than
₱18,500.

Solution:

80
Step 1: Draw a figure and represent the area.

P(x < 18,500)

Step 2. Find the z value for ₱18,500.

x ,500  2323,000


1818,500 ,000  4,500
z    1.80
 2,500 2,500

Step 3. Find the appropriate area. The area obtained in the Standardized
Normal Distribution Table is 0.4641, which corresponds to the area
between z = 0 and z = -1.80.

P 1.80 z  0   0.4641

Step 4. Subtract 0.4641 from 0.5000.

P (x < 18,500) = P (z < -1.80)

= 0.5000 – P (-1.80 < z < 0)

= 0.5000 - 0.4641

= 0.0359

0.0359

18,500 23,000

Hence, the probability that the employee owes less than ₱18,500 in PAG-
IBIG salary loan is 0.0359 or 3.59%.

Example 33: The average age of bank managers is 40 years. Assume the variable is
normally distributed. If the standard deviation is 5 years, find the
probability that the age of a randomly selected bank manager will be in
the range between 35 and 46 years old.

Solution: Assume that ages of bank managers are normally distributed; then cut off
points are as shown in the figure below.

Step 1. Draw a figure and represent the area.

81

35 40 46
Step 2. Find the two z values.

x 35  40  5 x 46  40 6
z    1.00 z    1.20
 5 5  5 5

Step 3. Find the appropriate area for z = -1.00 and z = 1.20 using the table.

P (-1.00 < z < 0) = 0.3413 P(0<z<1.20) = 0.3849

Step 4. Add P (-1.00 < z < 0) and P (0 < z < 1.20).

P (35 < x < 46) = P (-1.00 < z < 1.20)

= P (-1.00 < z < 0) + P (0 < z < 1.20)

= 0.3413 + 0.3849

= 0.7262

35 40 46 x - value
-1.00 1.20 z - value

Hence, the probability that a randomly selected bank manager is between 35


and 46 years old is 0.7262 or 72.62%.

Chapter
Exercises
Directions: Answer the following. Show all pertinent solutions.

82
1. Find the mean, median, and mode/modes of each of the following sets of
data:
a. 10, 12, 15, 16, 20, 25
b. 65, 73, 82, 76, 90, 32, 65, 70
c. 33, 45, 56, 39, 38, 33, 45, 54, 39, 32
d. 103, 234, 156, 365, 234, 268, 333, 103, 256, 365
e. 18, 24, 25, 16, 35, 21, 24, 33, 34, 25, 45,33,28, 17, 18, 16, 21, 45

2. The final grades of a student in six subjects where he was enrolled are shown
below. Find his/her grade point average.

Subject No. of Units (w) Grade (x)


Philosophy 3 2.5
Chemistry 5 1.5
English 3 2.25
Computer 2 3
Filipino 3 2.25
Calculus 6 2.5

3. Consider the following distribution below. Find the mean, median, and mode.

Class Interval f
80 - 89 8
70 - 79 15
60 - 69 29
50 - 59 45
40 - 49 39
30 - 39 31
20 - 29 19
10 - 19 9

4. The results of the midterm examination in Differential Calculus of BS Math


Junior students were taken and are presented in a frequency distribution.
Find the mean, median, and mode.

Class Interval f
94- 99 2
88- 93 7
82 - 87 19
76- 81 8
70- 75 10

83
64- 69 28
58- 63 37
52- 57 19
46- 51 8
40- 45 2

5. Find the range, average deviation, variance, and standard deviation of the
following sets of data:
a. 23, 21, 18, 17, 19, 21, 20, 18, 19, 24
b. 70, 65, 69, 73, 90, 87, 81, 89.
c. 24, 27, 32, 29, 31, 35, 27, 32, 23, 25, 30, 24.

6. The salaries of all the 130 employees of a company are tabulated in a


frequency distribution, as shown in the next page:

Salaries (in thousand Number of Workers


pesos)
33 - 36 4
29 - 32 10
25 - 28 12
21 - 24 24
17 - 20 38
13 - 16 18
9 - 12 15
5-8 6
1-4 3

Find the range, average deviation, variance, and standard


deviation

7. The test scores of 18 students in Analytic Geometry and Calculus I are as


follows: 27, 48, 33, 39, 52, 25, 50, 47, 42, 32, 21, 28, 42, 45, 55, 20, 37, and
38. Determine the following:
a. Q1 e. P27
b. P68 f. P68
c. Q3 g. P94.
d. D6 h. D7

8. The table below gives the age distribution of 100 individuals living in the
vicinity of Escolta.

Age Frequency
55 - 59 2
50 - 54 5

84
45 - 49 10
40 - 44 12
35 - 39 15
30 - 34 16
25 - 29 13
20 - 24 10
15 - 19 4
10 - 14 4

Solve for the following:


a. Q1 d. P85
b. Q3 e. P45
c. P10 f. D3

9. Find the area under the normal curve which lie:


a. Between z = -0.63 and z = 0.63
b. Between z = 0 and z = -1.25
c. To the right of z = -1.75
d. To the left of z = -1.30
e. To the left of z = 1.05

10. In a given distribution, the mean is 65 and the standard deviation is 6. Find
the corresponding standard score of:
a. 77
b. 47

11. For a certain type of computers, the length of time between charges of the
battery is normally distributed with a mean of 50 hours and a standard
deviation of 15 hours. John owns one of these computers and wants to know
the probability that the length of time will be between 50 and 70 hours.

85
Chapter 5:
Logic

Introduction
I
n everyday life, reasoning proves different points. For instance, to prove your
parents that you performed well in school, you can show your grades. To prove your
friends that you are a true friend to them, you just need to be a loyal and honest
friend. Similarly, mathematics and computer science use mathematical logic or
simply logic to prove results. In particular, mathematical logic is used in mathematics
to prove a theorem. In computer science, logic is used to prove results of computer
algorithm or the correctness of a computer program.

Logic is commonly referred as the science of correct reasoning, especially


regarding making inferences. Mathematical reasoning and arguments are based on
the rules of logic.

Learning Outcomes:
At the end of this chapter, you are expected to:

 Determine a given sentence as statement or not.


 Write compound statements using the language of logic.
 Create truth tables of given statements.
 Determine the truth value of a given statement.
 Write the converse, inverse, and contrapositive statements of conditional
statements.
 Determine whether statements are equivalent, tautologies or contradiction
using truth tables.
86
 Appreciate the nature and concept of logic as a tool to prove results.
LESSON 1:
LOGIC STATEMENTS AND QUANTIFIERS

Logic Statements

A statement is a declarative sentence that can be classified true or false, but


not both true and false. It may not be necessary to determine whether a sentence is
true to determine whether it is a statement or not.

Remember!
If we can determine the truth or falsity of a statement, then it has a
truth value. If the statement is true, then the truth value of the statement
is true. But when its false, then the truth value is false also.

Example 1: Identify Statements

Determine whether each sentence is a statement.

1. Annabelle is beautiful.
2. Where do you live?
3. 1+1=2
4. x + 5 = 7.

Solution:

1. For most people that knows Annabelle they might agree on the statement
but not all will have the same thought specially people who don’t like
Annabelle. Therefore, statement one is not a logical statement.
2. The sentence “Where do you live?” is a question; it is not a declarative
sentence. Hence, it is not a statement.
3. Everyone knows that if 1 is added to 1 the answer will always be 2. Then,
statement 2 is a logical statement.
4. x + 5 = 7 is a statement. It is known as an open statement, a sentence
that has a variable. The truth value of this statement will depends on the
value of x. It is true if x = 2, and it is false for any values of x. For any
given x, it is true or false but not both.

Simple and Compound Statements

87
A simple statement is a statement that conveys a single idea. A compound
statement is a statement that conveys two or more ideas. Connecting simple
statements with words and, or, if ...then, and if and only if creates a compound
statement.

Logic Connectives and Symbols


Symbolic Type of
Statement Connective
Form Statement
not p not ~p negation
p and q and p˄q conjunction
p or q or p˅q disjunction
If p, then q If ... then p→q conditional
p if and only if q If and only if p↔q biconditional

Remember!
The truth value of a simple statement is either true (T) or (F).

The truth value of a compound statement depends on the truth value of


its simple statements and connectives used.

A truth table is a table that shows the truth value of a compound


statement for all possible truth values of its simple statements.

Negation

The negation of a statement is the opposite of a statement. It is denoted with


either “not” or the symbol ~. For instance, if we like to negate a statement p its either
we use the symbol ~p or use not to indicate the negation. The negation gives the
opposite of the truth value.

Truth Table for Negation (p)


p ~p
T F
F T

Example 2: Negation of a Statement

Give the negation of each statement.

1. p: A square is a rectangle.
2. q: Today is Monday.

3. r: Every student has a ballpen.

Solution:

88
1. ~p: A square is not a rectangle.
2. ~q: Today is not Monday
3. ~r: Not every student has a ballpen

Example 3: Consider the following simple statements.

p: Today is Monday.
q: It is not raining.
r: I am going to a shopping.
s: I am not going to play volleyball game.

Write the following compound statements in symbolic form.

1. Today is Monday and it is raining.


2. It is not raining and I am going to a shopping.
3. I am going to play volleyball game or I am going to a shopping.
4. If it is raining, then I am not going to play volleyball game.
Solution:
1. p ˄ ~q
2. q˄r
3. ~s ˅ r
4. ~q → s

Example 4: Consider the following simple statements.

a: I am going to study
b: I am going to watch a movie.
c: I will not pass the exam.

Write the following compound statements in words.

1. ~a → c
2. b ˄ ~a
3. ~c ↔ a

Solution:
1. If I am not going to study, then I will not pass the exam.
2. I am going to watch a movie and I am not going to study.
3. I will pass the exam if and only if I am going to study.

Compound Statements and Grouping Symbols

If the compound statement is writtem symbolic form, then parentheses are


used to indicate which simple statements are grouped together.

Symbolic Form Meaning


p ˄ ( q ˅ ~ r) q and ~r are grouped together.

89
(p ˄ q) ˅ r p and q are grouped together.
(p ˄ q) → ( q ˅ ~ r) p and q are grouped together and q
and ~r are also grouped together.

If a compound statement is written in words, then a comma is used to


indicate which simple statements are grouped together. Statements on the same side
of a comma are grouped together.

English Sentence Meaning


p, and q or not r q and ~r are grouped together because
they are both on the same side of the
comma.
p and q, or r p and q are grouped together because
they are both on the same side of the
comma.
If p and r, then q or not r) p and q are grouped together because
they are both on the left side of the
comma.
q and ~r are also grouped together
because they are both on the right side
of the comma.

Example 5: Let p, q, and r represent the following statements:

p: Cathy’s dancing style is similar to Maja.


q: Cathy has a messy hair.
r: Cathy is a rock dancer.

1. Write (p ˄ q) → r as an english sentence.


2. Write “If Cathy is not a rock dancer, then Cathy does not have
messy hair and Cathy’s dancing style is not similar to Maja.” In
symbolic form.

Solution:
1. If Cathy’s dancing style is similat to Maja and Cathy has a messy
hair, then Cathy is a rock dancer.
2. ~ r → (~ q ˄ ~ p )

Conjunction

A conjunction is a compound statement that is made by combining two or


more statements with the word “and” or with symbol ˄.

Truth Table for Conjunction (p ˄ q)


p q p˄q
T T T

90
T F F
F T F
F F F

Remember!
The conjunction p ˄ q is true if and only if both p and q are
true.

Disjunction

A disjunction is a compound statement that is made by combining two or


more statements with the word “or” or with symbol ˄.

Truth Table for Disjunction (p v q)


P q p˅q
T T T
T F T
F T T
F F F

Remember!
The disjunction p ˅ q is true if and only if p is true, q is true, or
both p and q are true.

Example 6: Determine whether each statement is true or false.

1. 12 ≥ 10.
2. 6 is an even number and 6 is a composite number.
3. 5 is a prime number and 5 is an odd number.
4. 2 is an integer or 2 is an odd number.
5. – 5 ≤ - 10

Solution:

1. 12 ≥ 10 means 12 > 10 or 12 = 10. Since 12 > 10 is true, then the


statement is true.
2. The statement is true because 6 is an even number at the same time
composite number.
3. The statement is true because 5 is a prime and odd number.
4. The statement “2 is an integer” is true and the statement “2 is an odd
number” is false. Applying disjunction rule, the statement is true.
5. – 5 ≤ - 10 means - 5 < - 10 or -5 = -10. Since both simple statements are
false, the truth value of the statement is false.

91
LESSON 2:
TRUTH TABLES, EQUIVALENT STATEMENTS, AND
TAUTOLOGIES

Truth Table

A truth table is a table used to check the “truth value” of any compound
statement for all possible truth values of its simple statements. It is a way to check all
possible outcomes.
Presented below are the truth tables for negattion, conjunction, and
disjunction for review purposes.

Truth Table for Negation (p)


p ~p
T F
F T

Truth Table for Conjunction (p ˄ q)


p q p˄q
T T T
T F F
F T F
F F F

Truth Table for Disjunction (p v q)


p q p˅q
T T T
T F T
F T T
F F F

In this section we will construct truth tables for a statement that involves a
combination of negation, conjunctions and disjunctions. If the given statement
involves only two simple statements, then we have to start by constructing a table
with four rows, called the standard table form.

Example 7. Construct a table for ~ (p ˅ q) ˄ q

Solution: Start with the standard table form.

p q

92
T T
T F
F T
F F

Now, use the truth values of p and q to produce the truth value of p ˅ q

p q (p ˅ q)
T T T
T F T
F T T
F F F

Negate the truth values in the p ˅ q column to produce the truth value of
~ (p ˅ q)
P q p˅q ~ (p ˅ q)
T T T F
T F T F
F T T F
F F F T

Lastly, use the values of ~ (p ˅ q) and q to produce the truth values of


~ (p ˅ q) ˄ q applying the truth value of conjunction. The shaded column
is the truth table for ~ (p ˅ q) ˄ q.

P q p˅q ~ (p ˅ q) ~ (p ˅ q) ˄ q
T T T F F
T F T F F
F T T F F
F F F T F

Truth Table with Three or More Simple Statements

Compound statements that involve three or more simple statements require a


standard truth table form with 2n number of rows. For instance, if the statement
involves 3 simple statements, then the number of rows of truth table is 2 3 = 8. If 4
simple statements are involved, then 24 = 16 number of rows must be produced, and
so on.

Example 8. Construct a table for (p ˅ q) ˄ r.

Solution: Start with the standard truth table form with 3 simple statements involved.

P q r

93
T T T
T T F
T F T
T F F
F T T
F T F
F F T
F F F

Now, use the truth values of p and q to produce the truth value of p ˅
q

p q r (p ˅ q)
T T T T
T T F T
T F T T
T F F T
F T T T
F T F T
F F T F
F F F F

Lastly, use the values of (p ˅ q) and r to produce the truth values of


(p ˅ q) ˄ r applying the truth value of conjunction. The shaded column is
the truth table for (p ˅ q) ˄ r.

p q r (p ˅ q) (p ˅ q) ˄ r
T T T T T
T T F T F
T F T T T
T F F T F
F T T T T
F T F T F
F F T F F
F F F F F

Example 9. Construct a truth table for (p ˄ q) ˄ (~r ˅ q).

Solution: Using the procedure developed above, we can produce the following table.
Te shaded column is the truth table for p ˅ [~ (p ˄ ~q)]. The number below
signigfies the sequence in which columns were constructed.

94
P q r (p ˄ q) ~r (~r ˅ q) (p ˄ q) ˄ (~r ˅ q)
T T T T F T T
T T F T T T T
T F T F F F F
T F F F T T F
F T T F F T F
F T F F T T F
F F T F F F F
F F F F T T F
1 2 3 4

Equivalent Statements

Two statemens are said to be equivalent if they both have the same truth
value for all possible truth values of their simple statements. The symbol ≡ is used
to indicates equivalents statements.
Example 9. Show that ~ ( p ˅ ~ q) and ~p ˄ q are equivalent statements.

Solution: Construct the truth table of the given statements and compare the results.

For ~ ( p ˅ ~ q) :

P q ~q (p ˅ ~q) ~ (p ˅ ~q)
T T F T F
T F T T F
F T F F T
F F T T F

For ~p ˄ q:

P q ~p ~p˄q
T T F F
T F F F
F T T T
F F T F

Since the truth values of the given statements are the same, then
~(p ˅ ~ q) ≡ ~p ˄ q.

Tautologies and Contradiction

A tautology is a statement which is “always true” — that is, it is true for every
possible truth value. The opposite of a tautology is a contradiction, a statement
which is “always false”. In other words, a contradiction is false for every possible truth
value.

95
Example 10 . Show that (p → q) ∨ (q → p) is a tautology.

Solution: Construct the truth table for (p → q) ∨ (q → p) and show that the statement
is always true.

p q p→q q→p (p → q) ˅ (q → p)
T T T T T
T F F T T
F T T F T
F F T T T

The last column contains only T’s. Therefore, the statement is a tautology.

Example 11. Show that p ˄ (~p ˄ q) is a self – contradiction.

Solution: Construct the truth table for p ˄ (~p ˄ q) and show that the statement is
always false.

p q ~p ~p ˄ q p ˄ (~p ˄ q)
T T F F F
T F F F F
F T T T F
F F T F F

The last column contains only F’s. Therefore, the statement is a self -
contradiction.

LESSON 3:
THE CONDITIONAL AND THE BICONDITIONAL

Conditional Statements

A conditional statement is a statement written in a form “If...then”. The


phrase after the word “if” is called antecedent. The phrase after the word “then “is
called the consequent.

Example 12 . Determine the antecedent and consequent in the following statements.

1. If x + 5 = 8, then x = 3.
2. If a polygon hasfour sides, then it is a quadrilateral.
3. If you don’t study your lesson, then you will not pass the exam.
4. If 2 is an integer, then 2 is a rational number.

96
Solution:

1. Antecedent: x + 5 = 8
Consequent: x = 3

2. Antecedent: A polygon has four sides


Consequent: It is a quadrilateral

3. Antecedent: You don’t study your lesson


Consequent: You will not the pass the exam

4. Antecedent: 2 is an integer
Consequent: 2 is a rational number

The Truth Table for Conditional Statement ( p → q)


P q p→q
T T T
T F F
F T T
F F T

Remember!
The conditional p → q is false if p is true and q is false. It is true in
all other cases.

Example 13 . Determine the truth value of each statement.

1. If 2 is an integer, then 2 is a rational number.


2. If a=1, b=2, c=3, then a + b + c > 5
3. If 3 + 5 = 8, then 8 – 3 = 5.

Solution:

1. The consequent is true, then the statement is true.


2. The consequent is false, then the statement is false.
3. Because both the antecedent and consequent are true, then the
statement is true.

Binconditional Statements

The statement (p → q) ˄ (q → p) is called a biconditional and itt is denoted


by p ↔ q, which is read as “p if and only if q”.

97
The Truth Table for Biconditional Statement ( p ↔ q)
p q p↔q
T T T
T F F
F T F
F F T

Example 14 . Determine the truth value of each statement.

1. y + 7 = 12 if and only if y = 5
2. x2 = 25 if and only if x = 5
3. x > 4 if and only if x > 3.

Solution:
1. The statement is true since the statement “y + 7 = 12” will be only true if “y
= 5”. Otherwise, the statement will be false.
2. If x = -5, the first equation is true and the second equation is false. Thus,
the statement is false.
3. The statement is false. If x = 4, the first statement will become false.

LESSON 4:
THE CONDITIONAL AND RELATED STATEMENTS

Every conditional statement has three related statements. They are called the
converse, inverse, and contrapositive.

The Converse

Converse statement is a conditional statement that is formed by switching


the antecedent and the consequent of the conditional.

Remember!
The Converse of Conditional Statement

From p → q, converse statement will have q → p

The Inverse

An inverse statement is formed by negating both the antecedent and the


consequent of the conditional.

Remember!
98
The Inverse of Conditional Statement

From p → q, inverse statement will have ~p → ~q

The Contrapositive

Contrapositive is a conditional statement formed by negating both


antecedent and the consequent, and switching them. Contrapositive statements are
inverse and converse at the same time on a single conditional statement.

Remember!
The Inverse of Conditional Statement

From p → q, inverse statement will have ~q → ~p

Example 14: Write the converse, inverse, and contrapositive of the following
conditional statements.

1. If I sleep early, then I will wake up on time.


2. If today is Monday, then Kevin will play soccer.

Solution:
1. Converse: If I will wake up on time, then I sleep early.
Inverse: If I will not sleep early, then I will not wake up on time.
Contrapositive: If I will not wake up on time, then I will not sleep early.

2. Converse: If Kevin will play soccer, then today is Monday.


Inverse: If today is not Monday, then Kevin will not play soccer.
Contrapositive: If Kevin will not play soccer, then today is not Monday.

Truth Tables for Conditional and Related Statements

Conditional Converse Inverse Contrapositive


p q
p→q q→p ~p → ~ q ~q → ~p
T T T T T T
T F F T T F
F T T F F T
F F T T T T

99
Observations:

1. Converse and inverse statements are equivalents because they have the
same truth values.
2. Conditional and contrapositive statements are equivalents because they
have the same truth values.

Example 15: Determine whether related statements are equivalent.

1. If a number ends with 5, then the number is divisible 5.


If a number is divisible 5, then the number ends with a 5.

2. If two lines in a plane do not intersect, then the lines are parallel.
If two lines in a plane are not parallel, then the lines intersct.

Solution:

1. The second statement is the converse of the first. Thus, the statements
are not equivalent.
2. The second statement is the contrapositive of the first. Thus, the
statements are equivalent.

Chapter
Exercise
Directions: Answer the following as instructed.

24. Determine whether each sentence is a statement or not.

a. December is the first month of the year.


b. The diameter of a circle is twice the length of the radius.
c. He is very short.
d. The sums of the squares of the legs of a right triangle are equal to the
square of the hypotenuse.
e. 10 × 2 + 7 = 90.
f. A hexagon is a six-sided polygon.
g. Have a fun trip.
h. Do you like to swim?
i. Run!
j. 10 ≥ 10.

25. Write the negation of each statement and determine the truth value.

100
a. The sun rises from the east.
b. 1+1=0
c. January is the first month of the year.
d. The Eiffel Tower is in Japan.
e. A square has 7 sides.
f. A snake is a reptile.
g. The measure of an acute angle is greater than 90°.
h. 5 + 5 + 5 = 555
i. A quarter of an hour is 15 minutes
j. b) There is caffeine in coffee.

26. Write each symbolic statement in words. Use p, q, r, s, t, and u as defined


below.

p: The tour goes to Italy.


q: The tour goes to Spain.
r: We go to Venice.
s: We go to Florence.
t: The hotel fees are included.
u: The meals are not included

a. p ˄ ~q
b. r → ~s
c. s ↔ ~r
d. r˅s
e. p→r
f. ~t ˄ u

27. Write each sentence in symbolic form. Use p, q, r, and s as defined below.

p: Dwyane Wade is a football player.


q: Dwyane Wade is a basketball player.
r: Dwyane Wade is a rock star.
s: Dwyane Wade plays for the Miami Heat.

a. Dwyane Wade is a football player or a basketball player.


b. Dwyane Wade is a rock sta, and he is not a basketball player or a
football player.
c. If Dwyane Wade is a basketball player and a rock star, then he is not
a football player.
d. Dwyane Wade is a basketball player, if and only if he is not a football
player and he is not a rock star.
e. If Dwyane Wade plays for the Miami Heat, then he is a basketball
player and he is not a football player.
It is not true that, Dwyane Wade plays is a football player or a rock
star.

28. Construct a truth table for each compound statement.

101
a. p ˅ ~q
b. ~p ˄ q
c. (q ˄ ~p) ˅ ~q
d. ~(p ˅ q) ˄ (~r ˅ q)
e. [(p ˄ q) ˅ (r ˄ ~q)] ˄ (r ˅ ~q)

29. Identify the Hypothesis and Conclusion:

a. If Patsy has a messy hair today, then she will not show up to work.
b. If two lines are parallel, then they will not intersect.
c. If a polygon has 3 sides, then it is a triangle.
d. If the dog is barking, then someone is nearby.

30. Write each statement in "if, then" form:

a. The sum of the angles of a 3-sided polygon is 180 degree.


b. In a clear night, we can see the moon.
c. Getting a bachelor's degree will get you a job.

31. Determine the truth value of the following conditional and biconditional
statements.

a. If a ≤ 3, then a > 3.
b. If two lines intersect, then it must create a right angle
c. If a=1, b=2, c=3, then a + b + c > 5
d. If a polygon has 3 sides, then it is a triangle.
e. x = 5 if and only if x + 7 = 12.
f. If x > 5 if and only if x > 2.
g. The triangle is an isosceles, if and only if two sides are equal.
h. The angle is acute, if and only if the angle is less than 90° degrees.

32. Given the statements, write the inverse, converse, and contrapositive:

a. Two intersecting lines create an angle.


b. If today is Monday, then Kevin will play soccer.
c. If 1 + 2 = 3, then 12 + 22 = 32
d. If the polygon is a triangle, then it has 3 sides.
e. If x + 7 = 13, then x = 6.
f. If 3 is odd, then 3 + 1 is even.
g. If 2 is an integer, then 2 is a whole number.

102
Chapter 6:
Apportionment and Voting

Introduction
N
owadays, decision making is very vital. There are many situations where
decisions have to be made. For instance, when a family decides which house to buy,
every member of the family is given the chance to say something regarding the
choices. During national and local elections, people decides who will govern the
country, province, municipality, or barangay. Selecting baseball players to play in a
game, best performer to receive an award, and even deciding where to go for a
summer vacation, or deciding foods to eat on special occasions. All of these needs
sound decision making. There are many ways of making the final decisions, some
simple, some more complex.

In this chapter we will look at several voting methods, weighted voting


systems,and how votes are apportioned or divided among voters or states.

Learning Outcomes:
At the end of this chapter, you are expected to:

 Solve problems involving the various systems of:


a. Apportionment
b. Voting
c. Weighted Voting
 Appreciate the importance of apportionment, voting, and weighted voting
systems.
103
LESSON 1:
INTRODUCTION TO APPORTIONMENT

Apportionment is a method of dividing a whole into a various part. It started


in the year 1970 where the US House of Representatives agreed the number of
voters that would be represented by each member using apportionment methods.
The two competing plans were put forward by Alexander Hamilton and Thomas
Jefferson.

The Hamilton Plan

Using Hamilton Plan, the first step is to calculate the standard divisor. This
is the ratio of the total population to the number of people to apportion.

Remember!
total population
Standard Divisor ( SD)= apportion ¿
number of people ¿

The next step is to find the standard quota for each state. It is the whole
number part of the quotient of a population divided by the standard divisor. For
example, both 15.3 and 15.9 would be rounded to 15.

Example 1. Andromeda, a small country consisting of 6 states is governed by 25


representatives. The number of representatives for each state is
proportional to the population of the state. The following table shows the
population of each state as to the last census.

State Population
Alpha 22,000
Beta 45,000
Gamma 36,000
Delta 15,000
Epzilon 75,000
Zeta 41,000
Total 234,000

104
Solution: Find the standard divisor and the standard quotas for each of the states of
Andromeda.

Standard Divisor:

234,000
SD= =9 , 360
25

This means that each senate corresponds to a population of 9480 people.

Standard
State Population Quotient
Quota
22,000
Alpha 22,000 ≈ 2.35 2
9,360
45,000
Beta 45,000 ≈ 4.81 4
9,360
36,000
Gamma 36,000 ≈ 3.85 3
9,360
15,000
Delta 15,000 ≈ 1.60 1
9,360
75,000
Epzilon 75,000 ≈ 8.01 8
9,360
41,000
Zeta 41,000 ≈ 4.38 4
9,360
Total 234,000 22

From the calculations in the above table, the total number of representatives
is 22, not 25 as required. With this, the Hamilton plan calls for assigning an
additional representative to the state with the largest decimal remainder. This
process is continued until the number of representatives equals the number
required. Hence,

Standard Number of
State Population Quotient
Quota Senators
22,000
Alpha 22,000 ≈ 2.35 2 2
9,360
45,000
Beta 45,000 ≈ 4.81 4 5
9,360
36,000
Gamma 36,000 ≈ 3.85 3 4
9,360
15,000
Delta 15,000 ≈ 1.60 1 2
9,360
75,000
Epzilon 75,000 ≈ 8.01 8 8
9,360

105
41,000
Zeta 41,000 ≈ 4.38 4 4
9,360
Total 234,000 22 25

Overall, Alpha, Beta, Gamma, Delta, Epzilon, and Zeta gets 2, 5, 4, 2, 8, and
4 representatives, respectively.

The Jefferson Plan

The above method does not always yield the correct number of
representatives. In the last example, we were short for 3 representatives. The
Jefferson plan attempts to overcome this difficulty by using a modified standard
divisor. This number however is chosen by trial and error method until the sum of the
standard quotas met.

To do the Jefferson plan, the following steps must be considered:

1. Find the standard divisor.


2. Pick a modified divisor, d, that is slightly less than the standard divisor.
3. Divide each state’s population by the modified divisor to get its modified
quota.
4. Round each modified quota down to its lower quota.
5. Find the sum of the lower quotas.
6. If the sum is the same as the number of representatives to be
apportioned, you are done. If the sum is too large, pick a new modified
divisor that is larger than d. If the sum is too small, pick a new modified
divisor that is smaller than d. Repeat steps three through six until the
correct number of seats are apportioned.

Example 2. Using the problem in example 1, determine the number of


representatives for each state using Jefferson plan.

Solution: The standard divisor is 9360. Hence, choose a modified divisor less than
9360. Let say, 8300.

Standard
State Population Quotient
Quota
22,000
Alpha 22,000 ≈ 2.65 2
8,300
45,000
Beta 45,000 ≈5.42 5
8,300
36,000
Gamma 36,000 ≈ 4.34 4
8,300
15,000
Delta 15,000 ≈ 1.81 1
8,300

106
75,000
Epzilon 75,000 ≈ 9.04 9
8,300
41,000
Zeta 41,000 ≈ 4.93 4
8,300
Total 234,000 25

With Jefferson method, Alpha gets two representatives, Beta gets five
representatives, Gamma gets four representatives, Delta gets one
representative, Epsilon gets nine, and Zeta gets four representatives.

Huntington – Hill Apportionment Method

This method has been used by US Congress since 1940 to apportioned the
members of the Congress among the states. Huntington – Hill method is
implemented by calcultating what is called a Huntington – Hill number. This number
can be computed by

Remember!
(P A )2
,where P A is the population of state A and a is the
a(a+1)
current number of representatives from state A

When Huntington – Hill method is used to appotion representatives between


two states, the state with the higher Huntington – Hill number receives the next
representative.

Example 3. The table below shows the number of lifeguards that are assigned to
three sites of a certain beach resort and the number of rescues made by
lifeguards at those sites. Use the Huntington – Hill method to determine
to which site a new lifegurd shoud be assigned.

Number of Number of
Beach Sites
Lifeguards Rescues
A 25 1225
B 32 1516
C 18 975

Solution: Calculate the Huntington – Hill number for each beach site. In this
problem, the population is the number of rescues and the number of
representatives is the number of lifeguards.

Site A

107
2
(P A ) (1225)2
= ≈ 2,308.65
a(a+1) 25(25+ 1)

Site B

(PB )2 (1516)2
= ≈ 2,176.38
a(a+1) 32(32+1)

Site C
2
(P C ) (975)2
= ≈ 2,779.61
a(a+1) 18(18+1)

Hence, the new lifeguard should be assigned to Beach Site C


according to the Huntington – Hill Apportionment Principle.

LESSON 2
INTRODUCTION TO VOTING

Voting, in a mathematical perspective, is the process of aggregating the


preferences of individuals in a way that attempts to describe the preferences of a
whole group (Brilliant, 2020). Hence, voting is method for a group to make a
collective decision or express opinion. In this topic, we will discuss four different
voting systems.

Plurality Method of Voting

This is the most common method to determine the winner in an election. Each
voter votes for one candidate, and the candidate with the most votes wins. The
winning candidate does not have to have a majority of the votes.

Example 4. Sixty people were asked to rank their preferences of five fruits using 1
for the favorite and 5 for their least favorite. The results are shown in the
table below.

Fruits Rankings
Mango 1 2 1 3 2
Grapes 2 1 3 2 4
Apple 3 3 5 1 5
Orange 4 5 2 4 1
Durian 5 4 4 5 3

108
Number of Voters 15 17 12 9 7

According to this table, which fruit would win using the plurality voting
system?

Solution: To answer the question, we will make a table showing the number of first
place votes for each fruit.

Fruits First – place votes Total


Mango 15 + 12 27
Grapes 17 17
Apple 9 9
Orange 7 7
Durian 0 0

Because mango received 27 first - place votes, this type of fruit would
win the plurality test.

Borda Count Method of Voting

The problem with plurality voting is that alternative choices are not
considered. It focused only to the first choice. Second choice, third choice, and other
choices are disregarded. Unlike, plurality method, Borda count method considers all
choices by giving each choice a certain number of points.

Voters in Borda count method rank candidates from most to least favorable. Each
last-place vote is awarded one point; each next-to-last-place vote is awarded two
points, each third-from-last-place vote is awarded three points, and so on. The
candidate who receives the most points is the winner.

Example 5. The students of BSEd – Math 2 are going to elect a president from 4
nominees using the Borda count method. If the 60 students mark their
ballots as shown in the table below, who will be elected president?

Fruits Rankings
James 2 1 3 4 2
Aljon 1 2 2 1 3
Apple 3 3 4 2 1
Carla 4 4 1 3 2
Number of Voters 15 17 12 9 7

Solution: Using the Borda count method, each first-place vote receives 4 points,
each second – place vote receives 3 points, 2 points for each third – place
vote and 1 point for fourth – place vote. The summary for each candidate is
shown below.

109
James:

17 first – place votes 17 * 4 = 68


22 second – place votes 22 * 3 = 66
12 third – place votes 12 * 2 = 24
9 fourth – place votes 9*1 = 9
Total 167
Aljon:

24 first – place votes 24 * 4 = 96


29 second – place votes 29 * 3 = 87
7 third – place votes 7 * 2 = 14
0 fourth – place votes 0*1 = 0
Total 197
Apple:

7 first – place votes 7*4 = 28


9 second – place votes 9*3 = 27
32 third – place votes 32 * 2 = 64
12 fourth – place votes 12 * 1 = 12
Total 131
Carla:

12 first – place votes 12 * 4 = 48


7 second – place votes 7*3 = 21
9 third – place votes 9*2 = 18
32 fourth – place votes 32 * 1 = 32
Total 119

Using Borda count method, Aljon is the clear winner since he received the
highest total points.

Plurality with Elimination

As you have seen in the example of Plurality method, it is possible to produce


a winner even if majority of the voters did not vote for the winner. This situation can
be resolved by eliminating the candidates with the lowest votes and then holding
another election. If another election is done, naturally the supporters who voted for
the eliminated candidate will have to vote for their next choice. If the result does not
produce a majority winner, then the process of elimination is repeated until such time
the winner had win by majority votes.

Example 6. A certain college is to implement a new logo. Members were asked to


make designs and submit them for voting. Among the five designs
submitted, the 400 students are asked to rank the designs under
consideration. The results are shown in the following table.

Logo Rankings

110
Design 1 2 1 2 5 4
Design 2 1 2 3 2 5
Design 3 4 3 4 1 3
Design 4 3 5 1 4 2
Design 5 5 4 5 3 1
Number of Voters 47 70 108 80 95

Use the plurality with elimination method to determine which of these


designs should be adapted.

Solution: Because Design 2 received the lowest number of first – place votes, it is
eliminated. The new preference schedule is shown below

Logo Rankings
Design 1 1 1 2 4 4
Design 3 3 2 3 1 3
Design 4 2 4 1 3 2
Design 5 4 3 4 2 1
Number of Voters 47 70 108 80 95

From the table, Design 1 has 117 first – place votes, Design 3 has 80 first -
place votes, Design 4 has 108 first – placed votes, and Design 5 has 95
first – place votes. Because Design 3 has the fewest first – place votes, it
is eliminated from consideration. The new preference schedule is shown
below.

Logo Rankings
Design 1 1 1 2 3 3
Design 4 2 3 1 2 2
Design 5 3 2 3 1 1
Number of Voters 47 70 108 80 95

It can be noted that Design 1 has 117 first -place votes, Design 4 has 108
first place votes, and Design 5 has 175 first – place votes. From this,
Design 4 must be eliminated. The new preference schedule is shown
below.

Logo Rankings
Design 1 1 1 1 2 2
Design 5 2 2 2 1 1
Number of Voters 47 70 108 80 95

From this table, Design 1 received 225 first – place votes and Design 5 has
175 first – place votes. Since Design 1 received a majority votes, therefore,
Design 1 should be adapted.

111
Pairwise Comparison Voting Method

This voting method is sometimes called as the “head – to – head” method.


Which means each candidate is compared one – on – one with each of the other
candidates. Using this method, a winner candidate receives 1 point, 0.5 point for a
tie, and point nothing for a loss. The candidate garnered the highest number of points
wins the election.

Example 7. A certain company decided to have a team building activity. There are
four proposed beaches as the venue of the activity: Canhugas, Rawis,
Minasangay, and Calicoan. The preference schedule cast by 150
employees is shown below.

Beach Resort Rankings


Canhugas 2 1 3 4 3
Rawis 4 3 1 2 4
Minasangay 3 4 2 1 2
Calicoan 1 2 4 3 1
Number of Voters 32 40 25 15 38

Use the pairwise comparison voting method to determine which beach


the company should choose.
Solution: Let us create a table to keep track of each of the head – to – head
comparisons. This is shown below.

versus Canhugas Rawis Minasangay Calicoan


Canhugas
Rawis
Minasangay
Calicoan

To complete the table, we will place the name of the winner in the cell of each
head – to – head match.

 Canhugas versus Rawis

Canhugas was favored over Rawis on 32 + 40 + 38 = 110 votes


Rawis was favored over Canhugas on 25 + 15 = 40 votes

 Canhugas versus Minasangay

Canhugas was favored over Minasangay on 32 + 40 = 72 votes


Minasangay was favored over Canhugas on 25 + 15 + 38 = 78 votes

 Canhugas versus Calicoan

Canhugas was favored over Calicoan on 40 + 25 = 65 votes

112
Calicoan was favored over Canhugas on 32 + 15 + 38 = 85 votes

 Rawis versus Minasangay

Rawis was favored over Minasangay on 40 + 25 = 65 votes


Minasangay was favored over Rawis on 32 + 15 + 38 = 85 votes

 Rawis versus Calicoan

Rawis was favored over Calicoan on 25 + 15= 40 votes


Calicoan was favored over Rawis on 32 + 40 + 38 = 110 votes

 Minasangay versus Calicoan

Minasangay was favored over Calicoan on 25 + 15 = 40 votes


Calicoan was favored over Minasangay on 32 + 40 + 38= 110 votes

versus Canhugas Rawis Minasangay Calicoan


Canhugas Canhugas Minasangay Calicoan
Rawis Minasangay Calicoan
Minasangay Calicoan
Calicoan

It can be noted that, Calicoan has three wins, Minasangay has two wins, and
Canhugas has one win. Hence, Calicoan is the winning beach.

LESSON 3:
WEIGHTED VOTING SYSTEMS

A weighted voting system is one in which some voters have more weight on
the outcome of an election. Sometimes a player with a few votes can have as much
power as a player with many more votes or two players have almost an equal
number of votes, and yet one player has a lot of power and the other one has none.
A few examples are stockholders of a company, the United Nations Security
Council, and the European Union.

The number of votes that are required in weighted voting system to pass a
measure is called quota. The weight of a voter is the number of votes controlled by
the voter. For instance,

113
{25 :5 , 12 ,15 , 8 }

In the example above, 25 is the quota and the four numbers after the colon
are the weight of the voters which also indicates that there a total of four voters in this
system. Hence, the weight of Voter 1 is 5, the weight of Voter 2 is 12, the weight of
Voter 3 is 15, and the weight of Voter 4 is 8.

Four various weighted voting systems:

1. One person, one vote: For instance, {4: 1, 1, 1, 1, 1, 1, 1, 1}. In this system,
each person has one vote and five votes are needed to pass the measure.

2. Dictatorship: For example, {30: 31, 6, 8, 4, 5, 2}. In this system, the voter
with 31 votes can pass the measure. Even if the remaining five people get
together, their votes do not total the quota of 30.

3. Null system: For instance, {25: 3, 5, 6, 4}. In this system, if all number of
votes of four people will be added, the total number of votes would not sum to
the quota of 25. Therefore, no measure can be passed.

4. Veto power system: For example, {20: 5, 3, 2,10}. In this case, the sum of all
the votes is 20, the quota. Therefore, if any one voter does not vote for the
measure, it will fail. Each voter is said to have veto power.

Coalition

In a weighted voting system, a coalition is any set of players who might join
forces and vote the same way. In principle, we can have a coalition with as few as
one player and as many as all players. The coalition consisting of all the players is
called the grand coalition. A winning coalition is a set of voters whose sum of
votes is greater than or equal to the quota. A losing coalition is a set of voters
whose sum of votes is less than the quota. A voter who leaves a winning coalition
and thereby turns it into a losing coalition is called a critical voter.

There are many possible number of coalitions. This can be computed by


using the formula:

Remember!

n
Number of Possible Coalitions of n voters=2 −1

Example 8. Company J is owned by four individuals namely: James, Jerick,


Jonathan, and Jeff. James has 250 shares, Jerick has 375 shares,
Jonathan has 200 shares, and Jeff has 175 shares. The quota to pass a
measure is 501. The weighted voting system for this company is {501:

114
250, 375, 200, 175}. Determine the winning coalitions and the critical
voters for each winning coalition.

Solution: A winning coalition must have at least 501 votes. The winning coalitions
and critical voters for each winning coalition are:

Winning
Number of Votes Critical Voters
Coalitions
{A, B} 625 A, B
{B, C} 575 B, C
{B, D} 550 B, D
{A, B, C} 825 B
{A, B, D} 800 B
{A, C, D} 625 A, C, D
{B, C, D} 750 B
{A, B, C, D} 1000 None

Banzhaf Power Index

The Banzhaf power index, derived by John F. Banzhaf III in 1965, determine
the power of a voter in a weighted voting system. The Banzhaf power index of voter
v, can be computed by

Remember!

number of ×voter v is a critical voter


Banzhaf Power Index BPI ( v)=
number of ×any voter is a critical voter

Example 9. In reference to Example 8, determine the Banzhaf power index for A, B,


C, and D.

Solution: The winning coalitions and critical voters for each winning coalition in
Example 8 are:

Winning
Number of Votes Critical Voters
Coalitions
{A, B} 625 A, B
{B, C} 575 B, C
{B, D} 550 B, D
{A, B, C} 825 B
{A, B, D} 800 B
{A, C, D} 625 A, C, D
{B, C, D} 750 B

115
{A, B, C, D} 1000 None

For BPI (A):

To find BPI (A), we look under the critical voters column and find that A is a
critical voter twice. The number of times any voter is a critical voter is 12 (A,
B, and C are critical voters twice and B is a critical voter six times). Thus,

2
BPI ( A)= =0.17
12

For BPI (B):


6
BPI (B)= =0.50
12

For BPI (C):


2
BPI (C)= =0.17
12

For BPI (D):


2
BPI (D)= =0.17
12

Hence, Voter B has a higher Banzhaf power index.

Chapter
Exercises
Direction: Answer the following. Show all pertinent solutions.

1. The Eastern Samar State University – Salcedo Campus purchased 100


computer sets that are to be apportioned among the four offices, namely:
College of Natural Sciences (CANS), College of Education (COED), College
of Computer Science (CCS), and Graduate School. The table below shows
the enrollment for each office.

Office Enrollment
College of Agriculture and 450
Allied Sciences (CAAS)
College of Education 700
(COED)
College of Computer 500

116
Science (CCS)
Graduate School 120

a. Use the Hamilton method to determine the number of computers to be


apportioned to each office.
b. Use the Jefferson method to determine the number of computers to be
apportioned to each office.
c. How do the apportionment results produced using the Jefferson method
compare with the results produced using the Hamilton method?
d. If one additional computer set will be purchased, in what office it should
be placed?

2. A city has seven fire districts and 585 firefighters. The number of firefighters
assigned to each district is proportional to the population of the district. The
population for each district is given in the following table.

District Population
1 23,400
2 48,700
3 36, 500
4 32, 950
5 28, 450
6 18, 550
7 42,680
Total 231,230

a. Use the Hamilton method to apportioned the 585 firefighters.


b. Use the Jefferson method to apportioned the 585 firefighters.
c. How do the apportionment results produced using the Jefferson method
compare with the results produced using the Hamilton method?

3. A survey was conducted to 400 college freshmen to determine their favorite


sports. The students ranked the sports in order of preference; the results are
shown in the schedule below.

Sport Rankings
Basketball 1 2 5 4 2 1
Volleyball 3 3 2 5 1 5
Badminton 4 1 3 3 3 2
Baseball 2 5 4 1 5 4
Athletics 5 4 1 2 3 3
Number of Voters 65 85 45 70 75 60

a. Using plurality voting, which sport is the students’ favorite?

117
b. Using Borda count method, which sport is the students ’favorite?

4. A basketball team must choose the colors for its uniform. The coach offered
four different options, and the players ranked them in order of preference, as
shown in the table below.

Options Rankings
Red and White 1 4 2 3
Green and Yellow 3 2 1 2
White and Black 2 1 4 4
Blue and White 4 3 3 1
Number of Voters 4 2 4 5

a. Using the plurality with elimination method, what colors should be used for
the uniforms?
b. Using the pairwise comparison method, what colors should be used for
the uniforms?

5. Suppose the stock in a company is held by six people, Maritess, Virgie,


Antonette, Roselyn, Noeme, and Jocelyn. The voting system for this company
is {500: 75, 110, 90, 120, 95, 105}.

a. Determine the winning coalitions.


b. For each winning coalition, determine the critical voters.
c. Determine the Banzhaf power index for each voter.

118
References

Acelejado, M., Belecera, R., & Blay, B. (1999). Mathematics for the Millennium.
DIWA Scholastic Press Inc. Makati Philippines, Philippines.

Asaad, A., & Hailaya, W. (2001). Statistics as Applied to Education and Other
Related Fields. REX Book Store. Manila, Philippines.

Aufman, R., Lockwood, J., Nation, R., & Clegg, D. (2013). Mathematical Excursions,
3rd edition. Brooks/Cole, CENGAGE Learning.

Aufman, R., Lockwood, J., Nation, R., Clegg, D., & Epp, S. (2018). Mathematics in
the Modern World Philippine Edition. Rex Bookstore, Inc.

Aufmann R., Lockwood J, et al. (2018). Basic College Mathematics an Applied


Approach, 14th edition. Cengage Learning Asia: Philippines

Aufmann R., Lockwood J, et al. (2018). Mathematics in the Modern World,14th


edition. Cengage Learning Asia: Philippines

Bolaños, A. (1997). Probability and Statistical Concepts: An Introduction. REX Book


Store. Manila, Philippines.

Brilliant (2020). Mathematics of Voting. Retrieved from


https://brilliant.org/wiki/mathematics-of-voting/.

Calingasan, R., Martin, M., & Yambao, E. (2018). Mathematics in the Modern World.
Quezon City: C & E Publishing Inc.

Carpio J. and Peralta B. (2018). “Mathematics in the Modern World”. Books Atbp.
Publishing Corp.

Esller, B. et. al. (2012). Basic Statistics Textbook-workbook. Grand books Publishing
Inc. Manila, Philippines

119
Gabino, P. (2015). Statistics Concepts and Procedures.

Gabuyo, Y., & Dy, G. (2013). Assessment of Learning II Textbook and Workbook.
REX Book Store. Manila, Philippines

Pagala R. (2013). Basic Mathematics. Mindshapers Manila, Philippines.

PH Civil Service Reviewer (2014). How to Solve Venn Diagram Part 2. Retrieved
from http://civilservicereview.com/2014/12/solve-venn-diagram-problems-
part-2/

Shoenfield, J. R. (2018). Mathematical logic. CRC Press.

Sirug W. Ph.D. (2015). Basic Probability and Statistics. Mindshapers: Manila


Philippines

Sirug, W. (2018). Math in the Modern World. MINDSHAPERS Co., Inc. Manila,
Philippines

https://www.studypug.com/home.html

https://www.wright.edu/degrees-and/catalog/courses/mth - 1450

120

You might also like